Re: [obm-l] f(f(x))=e^(-x)

2023-09-24 Por tôpico Carlos Gustavo Tamm de Araujo Moreira
Por outro lado existem funções (necessariamente descontínuas) de R em R que
satisfazem essa equação funcional. Vou tentar
descrever uma delas.
Seja a=LambertW(1)~0,5671432904... a solução real de e^(-x)=x, como o Ralph
mencionou. Vou escrever g(x)=e^(-x).
Queremos f(f(x))=g(x). Vamos definir recursivamente g^n(x) por g^0(x)=x,
g^(n+1)(x)=g(g^n(x)). Vou usar o seguinte fato,
que deixo como exercício: para todo y real diferente de a existe um único x
em (-infinito,0] e um único n natural tais que y=g^n(x).
Definimos f(x) para x em (-infinito,-1] como f(x)=-(x+1)/x (assim f leva
(-infinito,-1] em (-1,0]), e definimos f em (-1,0] para
termos f(f(x))=g(x) se x está em (-infinito,-1], ou seja, f(y)=e^(1/(y+1))
para y em (-1,0]. A partir daí, se y=g^n(x) com n
natural e x em (-infinito,0], definimos f(y)=g^n(f(x)).Finalmente definimos
f(a)=a.
Abraços,
Gugu

On Sat, Sep 23, 2023 at 9:32 PM Ralph Costa Teixeira 
wrote:

> P.S.: Existe um argumento simples para mostrar que NÃO existe *f:R->R*
> *contínua* com f(f(x))=g(x) que serve para qualquer g estritamente
> decrescente (como esta g(x)=e^(-x)). Funciona assim:
>
> i) f teria que ser bijetiva. Afinal, f(a)=f(b) implica f(f(a))=f(f(b)) e,
> daqui (g bijetiva) vem a=b.
> ii) Mas f bijetiva continua em R implica f (estritamente) monótona!
> iiia) se f (estritamente) crescente, absurdo, pois f(f(x))=g(x) seria
> crescente;
> iiib) se f (estritamente) decrescente, absurdo, pois f(f(x))=g(x) seria
> crescente de novo!
>
> Ralph.
>
> On Sat, Sep 23, 2023 at 9:03 PM Ralph Costa Teixeira 
> wrote:
>
>> Tecnicamente esta f existe: você pode tomar f:{a}->{a} dada por f(a)=a
>> onde a=LambertW(1)~0,56714...  (a raiz de e^(-x)=x).  ;D ;D ;D
>>
>> Ou melhor dizendo: o problema fala algo sobre o domínio dessa f? Ou dela
>> ser contínua, pelo menos?
>>
>>
>> On Sat, Sep 23, 2023 at 8:25 PM Luís Lopes  wrote:
>>
>>> Saudações,
>>>
>>> Existe tal f? Se sim, qual seria?
>>>
>>> Recebi um e-mail com esta pergunta, sem maiores detalhes. Pelo e-mail,
>>> tal f não existe. Problema encontrado pelo remetente no YouTube.
>>>
>>> Luís
>>>
>>>
>>> --
>>> Esta mensagem foi verificada pelo sistema de antivírus e
>>>  acredita-se estar livre de perigo.
>>>
>>>
>>> =
>>> Instruções para entrar na lista, sair da lista e usar a lista em
>>> http://www.mat.puc-rio.br/~obmlistas/obm-l.html
>>> =
>>>
>>
> --
> Esta mensagem foi verificada pelo sistema de antivírus e
> acredita-se estar livre de perigo.

-- 
Esta mensagem foi verificada pelo sistema de antiv�rus e
 acredita-se estar livre de perigo.



[obm-l] Re: [obm-l] Uma recorrência diferente

2023-04-06 Por tôpico Carlos Gustavo Tamm de Araujo Moreira
Caro Vanderlei,
Não parece haver uma fórmula fechada muito simples. Veja
https://oeis.org/A85 para várias referências
sobre essa sequência.
Abraços,
Gugu

On Wed, Apr 5, 2023 at 11:41 PM Professor Vanderlei Nemitz <
vanderma...@gmail.com> wrote:

> Oi, mestres!
>
> Estava resolvendo um problema de combinatória e obtive essa recorrência:
>
> *x(n) = x(n - 1) + (n - 1).x(n - 2), com x1 = 1 e x2 = 2*.
>
> Por exemplo, x3 = x2 + 2.x1 e x9 = x8 + 8.x7
>
> Como resolver quando os coeficientes não são todos constantes?
>
> Apenas como curiosidade, o problema que originou a recorrência é:
> (IME - RJ) - Um professor dá um teste surpresa para uma turma de 9 alunos,
> e diz que o teste pode ser feito sozinho ou em grupos de 2 alunos. De
> quantas formas a turma pode ser organizar para fazer o teste? (Por exemplo,
> uma turma de 3 alunos pode ser organizar de 4 formas e uma turma de 4
> alunos pode se organizar de 10 formas)
>
>
>
> 
>  Não
> contém vírus.www.avast.com
> 
> <#m_4118911927943204904_DAB4FAD8-2DD7-40BB-A1B8-4E2AA1F9FDF2>
>
> --
> Esta mensagem foi verificada pelo sistema de antivírus e
> acredita-se estar livre de perigo.

-- 
Esta mensagem foi verificada pelo sistema de antiv�rus e
 acredita-se estar livre de perigo.



Re: [obm-l] Arimetica Diofanto

2022-02-01 Por tôpico Carlos Gustavo Tamm de Araujo Moreira
Vou enviar uma solução resumida:
Se 3^x-5^y=2, vamos testar os menores valores de y: se y=0 então 3^x=3 e
x=1. Se y=1 então 3^x=7, o que não dá solução inteira.
Se y=2 então 3^x=27 e x=3. Vamos então supor y=2+b>2, o que dá 3^x>27, logo
x=3+a>3, e podemos escrever a equação como
3^3(3^a-1)=5^2(5^b-1). Como a ordem de 3 módulo 5^2 é 20, segue que 20|a,
logo 3^a-1 é múltiplo de 3^20-1, e portanto é múltiplo de 61.
Como a ordem de 5 módulo 61 é 30, segue que 30|b, logo 5^b-1 é múltiplo de
5^30-1, e portanto é múltiplo de 31. Como a ordem de 3
módulo 31 é 30, segue que 30|a, logo 3^a-1 é múltiplo de 3^30-1, e portanto
é múltiplo de 271. Como a ordem de 5 módulo 271 é 27,
segue que 27|b, e como 30|b segue que 54|b, logo 5^b-1 é múltiplo de
5^54-1, e portanto é múltiplo de 81, absurdo, pois, como 3^a-1 não
é múltiplo de 3,  3^3(3^a-1) não é múltiplo de 81.
Abraços,
 Gugu
é múltiplo de 81

On Fri, Jan 28, 2022 at 5:28 PM Carlos Gustavo Tamm de Araujo Moreira <
g...@impa.br> wrote:

> Mas acho que lá uma solução está incompleta e as outras duas erradas...
>
> On Fri, Jan 28, 2022 at 5:11 PM Gabriel Torkomian 
> wrote:
>
>> https://artofproblemsolving.com/community/q1h2640462p22841017
>> Tem no aops
>>
>> Em sex., 28 de jan. de 2022 10:32, Israel Meireles Chrisostomo <
>> israelmchrisost...@gmail.com> escreveu:
>>
>>> 3^x-5^y=2
>>>
>>> Em sex., 28 de jan. de 2022 09:53, Esaú Gomes 
>>> escreveu:
>>>
>>>> E qual a equação?
>>>>
>>>> On Wed, Jan 26, 2022 at 3:33 PM Israel Meireles Chrisostomo <
>>>> israelmchrisost...@gmail.com> wrote:
>>>>
>>>>> Olá pessoal, recentimente estava estudando e me deparei com uma
>>>>> equação diofantina.eu tentei resolve-la mas ñ sei se está correta a
>>>>> solução ou incompleta, vcs poderiam por favor me ajudar a fechar o
>>>>> argumento?ñ quero outra solução só quero fazer da minha solução uma 
>>>>> solução
>>>>> top.Tenho a impressão que falta alguma coisa.
>>>>>
>>>>> --
>>>>> Esta mensagem foi verificada pelo sistema de antivírus e
>>>>> acredita-se estar livre de perigo.
>>>>
>>>>
>>>> --
>>>> Esta mensagem foi verificada pelo sistema de antivírus e
>>>> acredita-se estar livre de perigo.
>>>
>>>
>>> --
>>> Esta mensagem foi verificada pelo sistema de antivírus e
>>> acredita-se estar livre de perigo.
>>
>>
>> --
>> Esta mensagem foi verificada pelo sistema de antivírus e
>> acredita-se estar livre de perigo.
>
>

-- 
Esta mensagem foi verificada pelo sistema de antiv�rus e
 acredita-se estar livre de perigo.



Re: [obm-l] Arimetica Diofanto

2022-01-28 Por tôpico Carlos Gustavo Tamm de Araujo Moreira
Mas acho que lá uma solução está incompleta e as outras duas erradas...

On Fri, Jan 28, 2022 at 5:11 PM Gabriel Torkomian  wrote:

> https://artofproblemsolving.com/community/q1h2640462p22841017
> Tem no aops
>
> Em sex., 28 de jan. de 2022 10:32, Israel Meireles Chrisostomo <
> israelmchrisost...@gmail.com> escreveu:
>
>> 3^x-5^y=2
>>
>> Em sex., 28 de jan. de 2022 09:53, Esaú Gomes 
>> escreveu:
>>
>>> E qual a equação?
>>>
>>> On Wed, Jan 26, 2022 at 3:33 PM Israel Meireles Chrisostomo <
>>> israelmchrisost...@gmail.com> wrote:
>>>
 Olá pessoal, recentimente estava estudando e me deparei com uma equação
 diofantina.eu tentei resolve-la mas ñ sei se está correta a solução ou
 incompleta, vcs poderiam por favor me ajudar a fechar o argumento?ñ quero
 outra solução só quero fazer da minha solução uma solução top.Tenho a
 impressão que falta alguma coisa.

 --
 Esta mensagem foi verificada pelo sistema de antivírus e
 acredita-se estar livre de perigo.
>>>
>>>
>>> --
>>> Esta mensagem foi verificada pelo sistema de antivírus e
>>> acredita-se estar livre de perigo.
>>
>>
>> --
>> Esta mensagem foi verificada pelo sistema de antivírus e
>> acredita-se estar livre de perigo.
>
>
> --
> Esta mensagem foi verificada pelo sistema de antivírus e
> acredita-se estar livre de perigo.

-- 
Esta mensagem foi verificada pelo sistema de antiv�rus e
 acredita-se estar livre de perigo.



Re: [obm-l] Re: transcendencia

2021-04-02 Por tôpico Carlos Gustavo Tamm de Araujo Moreira
Por outro lado, se v é algébrico e u é algébrico sobre o corpo Q(v) então u
é algébrico.
O meu exemplo é um pouco "roubado": parece que b satisfaz a equação
(a^2-2)b+a(a^2-2)=0, mas, como
a^2-2=0, essa equação é identicamente nula...
Abraços,
 Gugu

On Fri, Apr 2, 2021 at 4:57 PM Israel Meireles Chrisostomo <
israelmchrisost...@gmail.com> wrote:

> Muito obrigado professor gugu
>
> Em sex, 2 de abr de 2021 16:00, Carlos Gustavo Tamm de Araujo Moreira <
> g...@impa.br> escreveu:
>
>> Não. Se a=sqrt(2) e b=pi então a^3+b.a^2-2a-2b=0, por exemplo.
>>
>> Em sex, 2 de abr de 2021 15:31, Israel Meireles Chrisostomo <
>> israelmchrisost...@gmail.com> escreveu:
>>
>>>   Se u é um número transcendente e v é um número, se u,v são
>>> algebricamente dependentes então v é transcendente?
>>>
>>>
>>> Em sex., 2 de abr. de 2021 às 14:58, Israel Meireles Chrisostomo <
>>> israelmchrisost...@gmail.com> escreveu:
>>>
>>>> Se a é um número transcendente e v é um número, se u,v são
>>>> algebricamente dependentes então v é transcendente?
>>>>
>>>> --
>>>> Israel Meireles Chrisostomo
>>>>
>>>
>>>
>>> --
>>> Israel Meireles Chrisostomo
>>>
>>> --
>>> Esta mensagem foi verificada pelo sistema de antivírus e
>>> acredita-se estar livre de perigo.
>>
>>
>> --
>> Esta mensagem foi verificada pelo sistema de antivírus e
>> acredita-se estar livre de perigo.
>
>
> --
> Esta mensagem foi verificada pelo sistema de antivírus e
> acredita-se estar livre de perigo.

-- 
Esta mensagem foi verificada pelo sistema de antiv�rus e
 acredita-se estar livre de perigo.



Re: [obm-l] Re: transcendencia

2021-04-02 Por tôpico Carlos Gustavo Tamm de Araujo Moreira
Não. Se a=sqrt(2) e b=pi então a^3+b.a^2-2a-2b=0, por exemplo.

Em sex, 2 de abr de 2021 15:31, Israel Meireles Chrisostomo <
israelmchrisost...@gmail.com> escreveu:

>   Se u é um número transcendente e v é um número, se u,v são
> algebricamente dependentes então v é transcendente?
>
>
> Em sex., 2 de abr. de 2021 às 14:58, Israel Meireles Chrisostomo <
> israelmchrisost...@gmail.com> escreveu:
>
>> Se a é um número transcendente e v é um número, se u,v são algebricamente
>> dependentes então v é transcendente?
>>
>> --
>> Israel Meireles Chrisostomo
>>
>
>
> --
> Israel Meireles Chrisostomo
>
> --
> Esta mensagem foi verificada pelo sistema de antivírus e
> acredita-se estar livre de perigo.

-- 
Esta mensagem foi verificada pelo sistema de antiv�rus e
 acredita-se estar livre de perigo.



[obm-l] Re: [obm-l] transcendência

2020-07-08 Por tôpico Carlos Gustavo Tamm de Araujo Moreira
Caro Israel,
Sim. Suponha que x e y são algebricamente dependentes sobre um corpo de
base K. Se y é algébrico, K(y)|K é uma extensão algébrica. Como x é raiz de
uma equação polinomial com coeficientes em K(y) (pois  x e y são
algebricamente dependentes), a extensão K(x,y)=K(y)(x)|K(y) é algébrica.
Portanto, como K(x,y)|K(y) e K(y)|K são extensões algébricas, K(x,y)|K
também é algébrica, e, em particular, x também é algébrico. Portanto, se x
é transcendente então y também é transcendente.
Abraços,
 Gugu


On Wed, Jul 8, 2020 at 2:04 PM Israel Meireles Chrisostomo <
israelmchrisost...@gmail.com> wrote:

> se dois números são algebricamente dependentes e se um deles é
> transcendente então isso implica o outro seja transcendente?isso me
> parece meio óbvio mas nao sei como provar
>
> --
> Israel Meireles Chrisostomo
>
> --
> Esta mensagem foi verificada pelo sistema de antivírus e
> acredita-se estar livre de perigo.

-- 
Esta mensagem foi verificada pelo sistema de antiv�rus e
 acredita-se estar livre de perigo.



[obm-l] Método para participar da IOI(International Olympiad in Informatics)

2020-07-07 Por tôpico Gustavo Bruno
Caro Senhor(a),

Eu sei que você deve estar muito ocupado e que recebe muitos emails,
portanto isso deve levar apenas sessenta segundos de leitura.

Eu obtive 2 medalhas de bronze na OBMEP nos últimos 2 anos, ambas de nível
2. Trabalhei como programador freelancer para uma empresa canadense e, como
consequência, possuo boas habilidades de programação e Inglês.

Gostaria de saber qual seria o melhor método para estudar e conseguir uma
medalha de ouro na OBM e, posteriormente, entrar na seleção para a IOI.
Acredito que estudar atráves dos bancos de questões e, periodicamente,
leitura de material complementar seja um bom método, mas gostaria de saber
a sua opinião.

Eu entendo se você estiver muito ocupado para responder, mas até mesmo uma
resposta de uma ou duas linhas me faria muito feliz,

Atenciosamente,

Gustavo

-- 
Esta mensagem foi verificada pelo sistema de antiv�rus e
 acredita-se estar livre de perigo.



[obm-l] Re: [obm-l] Re: [obm-l] Re: [obm-l] Re: [obm-l] Teoria dos números

2020-03-17 Por tôpico Carlos Gustavo Tamm de Araujo Moreira
Acho que a pergunta deve ser qual é o maior inteiro positivo que divide
essa expressão para todo valor de n ao mesmo tempo.

On Tue, Mar 17, 2020 at 6:58 AM Pedro José  wrote:

> Bom dia!
> Se você considerar a expressão n(427-90n-70n^2+45n^3+18n^4)
> D=|n(427-90n-70n^2+45n^3+18n^4)|
> Por exemplo, n=1
> D=330.
> Agora se liberar n para variar D tende a oo.
>
> Se n for raiz da expressão, também tende a oi, pois qualquer inteiro
> divide 0.
>
>
> Em seg, 16 de mar de 2020 22:16, Israel Meireles Chrisostomo <
> israelmchrisost...@gmail.com> escreveu:
>
>> não entendi
>>
>> Em seg., 16 de mar. de 2020 às 22:01, Pedro José 
>> escreveu:
>>
>>> Para um dado n é o módulo do valor da expressão.
>>>
>>> Em seg, 16 de mar de 2020 21:49, Pedro José 
>>> escreveu:
>>>
 Boa noite!
 O módulo dessa expressão tende a oo. Não existe máximo.
 Saudações,
 PJMS

 Em seg, 16 de mar de 2020 20:36, Israel Meireles Chrisostomo <
 israelmchrisost...@gmail.com> escreveu:

> Qual é o maior inteiro que divide  n (427 - 90n - 70n^2 + 45n^3 +
> 18n^4)?
>
> --
> Israel Meireles Chrisostomo
>
> --
> Esta mensagem foi verificada pelo sistema de antivírus e
> acredita-se estar livre de perigo.


>>> --
>>> Esta mensagem foi verificada pelo sistema de antivírus e
>>> acredita-se estar livre de perigo.
>>
>>
>>
>> --
>> Israel Meireles Chrisostomo
>>
>> --
>> Esta mensagem foi verificada pelo sistema de antivírus e
>> acredita-se estar livre de perigo.
>
>
> --
> Esta mensagem foi verificada pelo sistema de antivírus e
> acredita-se estar livre de perigo.

-- 
Esta mensagem foi verificada pelo sistema de antiv�rus e
 acredita-se estar livre de perigo.



[obm-l] Re: [obm-l] Re: [obm-l] Função Desconhecida

2019-12-20 Por tôpico Luiz Gustavo Alves Brandão
Como faz pra sair do grupo? Meu e-mail luizbg...@gmail.com.

Em sex., 20 de dez. de 2019 às 17:14, Luiz Antonio Rodrigues <
rodrigue...@gmail.com> escreveu:

> Olá, Esdras!
> Muito obrigado pela resposta!
> Vou fazer uma pesquisa sobre este assunto!
> Um abraço!
> Luiz
>
> Em sex, 20 de dez de 2019 4:38 PM, Esdras Muniz 
> escreveu:
>
>> Acho que essa função é trancendente.
>>
>> Em sex, 20 de dez de 2019 14:42, Luiz Antonio Rodrigues <
>> rodrigue...@gmail.com> escreveu:
>>
>>> Olá, pessoal!
>>> Tudo bem?
>>> Estou tentando, há alguns dias, resolver o seguinte problema:
>>>
>>> Preciso descobrir uma função f(x) cuja derivada é sen(x^3). Sabe-se que
>>> f(0)=2.
>>>
>>> Utilizei um software e mesmo assim não cheguei numa resposta para esta
>>> integral...
>>> Alguém sabe se esta função é de algum tipo "especial"?
>>> Muito obrigado!
>>> Luiz
>>>
>>> --
>>> Esta mensagem foi verificada pelo sistema de antivírus e
>>> acredita-se estar livre de perigo.
>>
>>
>> --
>> Esta mensagem foi verificada pelo sistema de antivírus e
>> acredita-se estar livre de perigo.
>
>
> --
> Esta mensagem foi verificada pelo sistema de antivírus e
> acredita-se estar livre de perigo.

-- 
Esta mensagem foi verificada pelo sistema de antiv�rus e
 acredita-se estar livre de perigo.



Re: [obm-l]

2019-11-16 Por tôpico Carlos Gustavo Tamm de Araujo Moreira
Oi pessoal,
Eu achava que sairia mais fácil olhando em Z[i.sqrt(2)], mas mesmo assim dá
trabalho. Há uma discussão bem mais completa sobre esse problema (que caiu
em uma olimpíada polonesa) em
https://mathoverflow.net/questions/250312/diophantine-equation-3n-1-2x2
Em particular há uma solução que envolve olhar uma recorrência (ligada à
equação de Pell) módulo 27 e módulo 17.
Abraços,
   Gugu

On Fri, Nov 15, 2019 at 5:17 PM Pedro José  wrote:

> Boa tarde!
> Esdras,
> Boa sacada!
> (b^2+1)^2=b^4+2b^2+1=b^4+(3^k)^2.
> Depois ternos pitagóricos sem restrição de primitivo.
> Aí subtraindo a primeira da segunda ou somando dão quadrados perfeitos em
> p e q. Basta igualar a1 ou então tira a raiz e iguala u^2 - v^2. Sai que
> p-q=1.
> Aí fica fácil.
> Parabéns!
> Falta achar uma lei de geração para outras soluções ou uma restrição
> (acredito mais nessa) para a e b ímpares.
> Saudações,
> PJMS
>
> Em sex, 15 de nov de 2019 13:05, Pedro José 
> escreveu:
>
>> Bom dia!
>> Esdras,
>> grato, vou tentar seguir a linha.
>>
>> Douglas,
>> Tentei combinar mod 8 com mod9 e não saiu uma restrição.
>>
>> Carlos Gustavo,
>> teria como propor material sobre o tema que você levantou. Compreendi a
>> fatoração, mas não como seriam os primos nesse universo.
>> Ainda sem tempo para tentar uma restrição.
>>
>> Saudações,
>> PJMS
>>
>>
>> Em ter., 12 de nov. de 2019 às 23:21, Esdras Muniz <
>> esdrasmunizm...@gmail.com> escreveu:
>>
>>> O caso "a" par eu fiz assim: a=2k, daí, (3^k)^2+ b^4=(d^2+1)^2, então vc
>>> usa que para algum par p, q, com 0>> p^2-q^2. Daí vc mostra que p=q+1 e em seguida que q=1.
>>>
>>> Em ter, 12 de nov de 2019 22:29, Prof. Douglas Oliveira <
>>> profdouglaso.del...@gmail.com> escreveu:
>>>
>>>> Será que não sai usando somente congruência módulo 8?
>>>>
>>>> Em ter., 12 de nov. de 2019 às 20:07, Pedro José 
>>>> escreveu:
>>>>
>>>>> Boa noite!
>>>>> Esdras,
>>>>> tem como você postar, mesmo para o caso apenas de n par?
>>>>>
>>>>> Grato!
>>>>>
>>>>> Saudações,
>>>>> PJMS.
>>>>>
>>>>> Em ter., 12 de nov. de 2019 às 19:52, Pedro José 
>>>>> escreveu:
>>>>>
>>>>>> Boa noite!
>>>>>> Carlos Gustavo,
>>>>>> grato pela luz, estava tão obsecado e só rodando em círculos, tal
>>>>>> qual patrulha perdida.
>>>>>>
>>>>>> Saudações,
>>>>>> PJMS
>>>>>>
>>>>>> Em ter., 12 de nov. de 2019 às 19:19, Esdras Muniz <
>>>>>> esdrasmunizm...@gmail.com> escreveu:
>>>>>>
>>>>>>> Dá para mostrar que a única solução com a e b pares é (2, 2). Agora
>>>>>>> com a e b ímpares, não consegui.
>>>>>>>
>>>>>>> Em ter, 12 de nov de 2019 18:19, Pedro José 
>>>>>>> escreveu:
>>>>>>>
>>>>>>>> Boa noite!
>>>>>>>> Agora captei vosso pensamento.
>>>>>>>> Só que ao transformar a equação em uma equação de Pell, nós
>>>>>>>> maculamos a função 3^n.
>>>>>>>> Em verdade a solução para a par a= 2n, seria (2,2); pois, como
>>>>>>>> mencionara anteriormente se a é par, b também o é.
>>>>>>>> Só que quando procuramos as outras soluções, baseando-se na
>>>>>>>> propriedade de que a norma em Q [RAiz(A)] conserva a multiplicação. Só 
>>>>>>>> que
>>>>>>>> quando eu pego a solução
>>>>>>>> 3 + 2 Raiz(2) e elevo ao quadrado 17 + 12 Raiz(2). Se eu pegar
>>>>>>>> 17^2-2*12^2=1 eu atendo x^2 - 2Y^2=1. E assim sucessivamente. Mas não
>>>>>>>> existe n inteiro tal que 3^n=17, então não é uma solução da equação
>>>>>>>> original.
>>>>>>>> Creio que seja um pouco mais complicada a solução. Pois o difícil é
>>>>>>>> saber quando atende também a 3^n.
>>>>>>>> Acredito que deva haver uma forma de restringir a essas soluções,
>>>>>>>> pois, definir em que condições a solução terá x como uma potência de 3 
>>>>>>>> seja
>>>>>>>> bem difícil.
>>>>>>>> Estou a

Re: [obm-l]

2019-11-12 Por tôpico Carlos Gustavo Tamm de Araujo Moreira
Há uma menção a esse problema em
https://math.stackexchange.com/questions/2826307/integer-solutions-of-3n-1-2m2
Uma sugestão é usar o fato de que Z[i.sqrt(2)] é um domínio de fatoração
única, e escrever 1+2b^2 como (1+b.i.sqrt(2))(1-b.i.sqrt(2)).
Notem que 3 se fatora aí como (1+i.sqrt(2))(1- i.sqrt(2)).
Abraços,
 Gugu

On Tue, Nov 12, 2019 at 7:21 PM Pedro José  wrote:

> Boa noite!
> Agora captei vosso pensamento.
> Só que ao transformar a equação em uma equação de Pell, nós maculamos a
> função 3^n.
> Em verdade a solução para a par a= 2n, seria (2,2); pois, como mencionara
> anteriormente se a é par, b também o é.
> Só que quando procuramos as outras soluções, baseando-se na propriedade de
> que a norma em Q [RAiz(A)] conserva a multiplicação. Só que quando eu pego
> a solução
> 3 + 2 Raiz(2) e elevo ao quadrado 17 + 12 Raiz(2). Se eu pegar
> 17^2-2*12^2=1 eu atendo x^2 - 2Y^2=1. E assim sucessivamente. Mas não
> existe n inteiro tal que 3^n=17, então não é uma solução da equação
> original.
> Creio que seja um pouco mais complicada a solução. Pois o difícil é saber
> quando atende também a 3^n.
> Acredito que deva haver uma forma de restringir a essas soluções, pois,
> definir em que condições a solução terá x como uma potência de 3 seja bem
> difícil.
> Estou apanhando mais do que mala velha em véspera de viagem.
> Se alguém postar uma solução, me ajudaria bastante.
>
> Saudações,
> PJMS
>
>
> Saudações,
> PJMS.
>
>
>
> Em ter., 12 de nov. de 2019 às 17:25, Pedro José 
> escreveu:
>
>> Boa tarde!
>> Douglas,
>> perdoe-me pela minha miopia, mas você poderia detalhar melhor onde entra
>> a equação de Pell?
>> A equação de Pell não é x^2-Dy^2 = N?
>> Se a é par b é par e se a ímpar b é ímpar para atender mod8,
>> Não consegui captar a sugestão.
>>
>> Saudações,
>> PJMS
>>
>> Em ter., 12 de nov. de 2019 às 16:50, Prof. Douglas Oliveira <
>> profdouglaso.del...@gmail.com> escreveu:
>>
>>> Hum, então, vamos analisar o caso de a ser par do tipo 2n.
>>>
>>> Assim podemos escrever que (3^n+b(sqrt2))(3^n-b(sqrt2))=1
>>> Dai através da solução mínima que o Pedro fez, como (1,1) por exemplo,
>>> da pra ver que são infinitas soluções usando a equação de Pell.
>>>
>>> Abraco
>>> Douglas Oliveira.
>>>
>>>
>>>
>>> Em dom, 10 de nov de 2019 19:33, gilberto azevedo 
>>> escreveu:
>>>
 [HELP]

 Achas todos os pares (a,b) inteiros positivos tais que :
 3^a = 2b² + 1.


 --
 Esta mensagem foi verificada pelo sistema de antivírus e
 acredita-se estar livre de perigo.
>>>
>>>
>>> --
>>> Esta mensagem foi verificada pelo sistema de antivírus e
>>> acredita-se estar livre de perigo.
>>
>>
> --
> Esta mensagem foi verificada pelo sistema de antivírus e
> acredita-se estar livre de perigo.

-- 
Esta mensagem foi verificada pelo sistema de antiv�rus e
 acredita-se estar livre de perigo.



[obm-l] Re: [obm-l] Re: [obm-l] Números eficientes

2019-08-30 Por tôpico Luiz Gustavo Alves Brandão
Se x é eficiente então x(x-1) é múltiplo de 1000. Como x e x-1 são
coprimos, um deles é 8A e o outro é 125B, com A e B inteiros e B ímpar.
Sendo assim, só é preciso testar B = 1, 3, 5 e 7, que nos fornece os
números eficientes 376 e 625.
Qualquer erro só avisarem...

Em sex, 30 de ago de 2019 às 14:52, Claudio Buffara <
claudio.buff...@gmail.com> escreveu:

> Achar estes números com uma planilha deve ser mais rápido do que fazer a
> análise usando congruências.
>
> On Fri, Aug 30, 2019 at 2:01 PM Carlos Monteiro <
> cacacarlosalberto1...@gmail.com> wrote:
>
>> Um número x de 3 algarismos é dito eficiente se os 3 últimos algarismos
>> de x^2 são os mesmos algarismos de x e na mesma ordem. Encontre todos os
>> números eficientes.
>>
>> --
>> Esta mensagem foi verificada pelo sistema de antivírus e
>> acredita-se estar livre de perigo.
>
>
> --
> Esta mensagem foi verificada pelo sistema de antivírus e
> acredita-se estar livre de perigo.

-- 
Esta mensagem foi verificada pelo sistema de antiv�rus e
 acredita-se estar livre de perigo.



[obm-l] Re: [obm-l] teoria da Medida - provar que f é contínua

2019-07-02 Por tôpico Carlos Gustavo Tamm de Araujo Moreira

Caro Artur,

Seja d>0 pequeno. Existem K compacto e U aberto com K C A C U e 
m(A)-d

(A interseção (x+A)) C (K interseção (x+K)) U (A\K) U (x+(A\K)), temos 
f(x)=m(A interseção (x+A))

m(K interseção (x+K))>f(x)-2d, para todo x em R^n.

Seja agora V aberto contendo (K interseção (x+K)) com m(V)Existe r>0 tq. |x-y|interseção (y+K)) C V,


donde f(y)=m((K interseção V) 
união ((y+K) interseção V))=m(K interseção V)+m((y+K) interseção V)-m(K 
interseção (y+K)),


logo, como m(K interseção V)>=m(K interseção (x+K))>f(x)-2d e m((y+K) 
interseção V)>=m((K interseção (x+K))+y-x)=m(K interseção 
(x+K))>f(x)-2d, temos


f(y)>=m(K interseção (y+K))>=m(K interseção V)+m((y+K) interseção 
V)-m(V)>=2f(x)-4d-m(V)>f(x)-5d. Isso dá a continuidade.


Abraços,

  Gugu

Em 02/07/2019 09:54, Artur Costa Steiner escreveu:
Será que alguém aqui pode me ajudar com isso, ou sabe de algum link 
correlato:?


Sejam  m a medida de Lebesgue, A um subconjunto de R^n com 0 < m(A) < 
oo e f: R^n ---> [0, oo) dada por f(x) =m(A intersecção (x + A)), onde 
x + A = {x + a | a está em A} a translação de A pelo vetor x de R^n.


Mostre que f é contínua.

Este teorema provê uma prova bem simples de que, se m(A) > 0, então A 
- A = {a1 - a2  | a1 e a2 estão em A}  contém uma bola com centro na 
origem. (conheço uma outra prova, que é mais trabalhosa), E esta 
conclusão sobre a bola é utilizada numa linda prova de que o conjunto 
de Vitali (que é bem patológico) não é Lebesgue mensurável.



Obrigado

Artur


--
Esta mensagem foi verificada pelo sistema de antiv�rus e
acredita-se estar livre de perigo. 


--
Esta mensagem foi verificada pelo sistema de antiv�rus e
acredita-se estar livre de perigo.



Re: [obm-l] Equação Funcional

2018-09-27 Por tôpico Claudio Gustavo
Olhei essa questão e achei interessante, pq a princípio parece simples mas 
depois vc empaca para achar o F(2)..Bom, o problema termina ao achar o F(2) e a 
ideia é usar o F(30) dado:F(30)=F(2.15)=F(2)+F(15)-1=F(2)+F(3)+F(5)-1-1 -> 
F(2)+F(3)+F(5)=6.Na lei inicial, encontramos facilmente F(0)=F(1)=1. Seja y=x: 
F(x^2)=2F(x)-1. Ou seja, como os únicos inteiros que ao quadrado resultam neles 
mesmo são 0 e 1, se houver pelo menos algum outro x tal que F(x)=1, então 
F(x^2) tb seria igual a 1 na relação acima e existiriam infinitos valores com 
imagem 2, portanto 0 e 1 são os únicos com essa imagem. Dessa forma F(x)=>2 
para valores naturais maiores que 1.Logo: F(2), F(3) e F(5) são maiores ou 
iguais a 2 e a soma seria maior ou igual a 6. Mas como descobrimos que a soma 
vale 6 e cada um é no mínimo 2, é fácil verificar que como o contradomínio são 
os naturais só resta a opção de todos os três valores serem iguais a 2! Sendo 
assim:F(14400)=2F(120)-1=2(F(30)+F(4)-1)-1=2(F(30)+2F(2)-1-1)-1=2F(30)+4F(2)-5 
-> F(14400)=11.
Abraços,Cláudio Gustavo.


Enviado do Yahoo Mail para iPhone


Em quarta-feira, setembro 19, 2018, 6:33 PM, Jeferson Almir 
 escreveu:

Peço uma ideia ou ajuda na seguinte questão:Sejam x e y naturais e uma função  
f : N -> N tais que F(xy) = F(x) + F(y) -1
Existe um número finito de numeros tais que F(x) = 1. 
F(30) = 4
Determine o F( 14400) 
--
Esta mensagem foi verificada pelo sistema de antiv�rus e 
 acredita-se estar livre de perigo.



-- 
Esta mensagem foi verificada pelo sistema de antiv�rus e
 acredita-se estar livre de perigo.



Re: [obm-l] Equação Funcional

2018-09-27 Por tôpico Claudio Gustavo
* com imagem 1


Enviado do Yahoo Mail para iPhone


Em quinta-feira, setembro 27, 2018, 7:48 AM, Claudio Gustavo 
 escreveu:

Olhei essa questão e achei interessante, pq a princípio parece simples mas 
depois vc empaca para achar o F(2)..Bom, o problema termina ao achar o F(2) e a 
ideia é usar o F(30) dado:F(30)=F(2.15)=F(2)+F(15)-1=F(2)+F(3)+F(5)-1-1 -> 
F(2)+F(3)+F(5)=6.Na lei inicial, encontramos facilmente F(0)=F(1)=1. Seja y=x: 
F(x^2)=2F(x)-1. Ou seja, como os únicos inteiros que ao quadrado resultam neles 
mesmo são 0 e 1, se houver pelo menos algum outro x tal que F(x)=1, então 
F(x^2) tb seria igual a 1 na relação acima e existiriam infinitos valores com 
imagem 2, portanto 0 e 1 são os únicos com essa imagem. Dessa forma F(x)=>2 
para valores naturais maiores que 1.Logo: F(2), F(3) e F(5) são maiores ou 
iguais a 2 e a soma seria maior ou igual a 6. Mas como descobrimos que a soma 
vale 6 e cada um é no mínimo 2, é fácil verificar que como o contradomínio são 
os naturais só resta a opção de todos os três valores serem iguais a 2! Sendo 
assim:F(14400)=2F(120)-1=2(F(30)+F(4)-1)-1=2(F(30)+2F(2)-1-1)-1=2F(30)+4F(2)-5 
-> F(14400)=11.
Abraços,Cláudio Gustavo.


Enviado do Yahoo Mail para iPhone


Em quarta-feira, setembro 19, 2018, 6:33 PM, Jeferson Almir 
 escreveu:

Peço uma ideia ou ajuda na seguinte questão:Sejam x e y naturais e uma função  
f : N -> N tais que F(xy) = F(x) + F(y) -1
Existe um número finito de numeros tais que F(x) = 1. 
F(30) = 4
Determine o F( 14400) 
--
Esta mensagem foi verificada pelo sistema de antiv�rus e 
 acredita-se estar livre de perigo.






-- 
Esta mensagem foi verificada pelo sistema de antiv�rus e
 acredita-se estar livre de perigo.



Re: [obm-l] Matrizes

2018-08-24 Por tôpico Claudio Gustavo
* identidade 


Enviado do Yahoo Mail para iPhone


Em sexta-feira, agosto 24, 2018, 10:55 AM, Claudio Gustavo 
 escreveu:

Adicione a indenidade aos dois lados da igualdade e obterá: (A+I)(B+I)=I.Logo, 
como uma é inversa da outra, comutam: (B+I)(A+I)=I.Daí: BA+A+B=0, logo AB=BA.
Abraços 

Enviado do Yahoo Mail para iPhone


Em terça-feira, agosto 21, 2018, 11:01 PM, Vanderlei Nemitz 
 escreveu:

Boa noite, pessoal!Resolvi a seguinte questão, mas de uma forma um tanto 
complicada.Gostaria de uma solução mais simples.Muito obrigado!Vanderlei
SejamA e B matrizes reais n x n tais que AB + A + B = 0. Prove que AB = BA.    

--
Esta mensagem foi verificada pelo sistema de antiv�rus e 
 acredita-se estar livre de perigo.






-- 
Esta mensagem foi verificada pelo sistema de antiv�rus e
 acredita-se estar livre de perigo.



Re: [obm-l] Matrizes

2018-08-24 Por tôpico Claudio Gustavo
Adicione a indenidade aos dois lados da igualdade e obterá: (A+I)(B+I)=I.Logo, 
como uma é inversa da outra, comutam: (B+I)(A+I)=I.Daí: BA+A+B=0, logo AB=BA.
Abraços 

Enviado do Yahoo Mail para iPhone


Em terça-feira, agosto 21, 2018, 11:01 PM, Vanderlei Nemitz 
 escreveu:

Boa noite, pessoal!Resolvi a seguinte questão, mas de uma forma um tanto 
complicada.Gostaria de uma solução mais simples.Muito obrigado!Vanderlei
SejamA e B matrizes reais n x n tais que AB + A + B = 0. Prove que AB = BA.    

--
Esta mensagem foi verificada pelo sistema de antiv�rus e 
 acredita-se estar livre de perigo.



-- 
Esta mensagem foi verificada pelo sistema de antiv�rus e
 acredita-se estar livre de perigo.



Re: [obm-l] Re: Polinomios simétricos

2018-04-28 Por tôpico Claudio Gustavo
Boa tarde!Vc pode isolar o z em cada expressão e usar multiplicador de 
lagrange:Z = 5 - (x+y) = (3-xy)/(x+y).
Logo:(-(3+x^2)/(x+y)^2 , -(3+y^2)/(x+y)^2) = k.(-1 , -1).Vc chega em: x=y ou 
x=-y(essa não convém).Daí: 3x^2 -10x+3=0, logo x=3 ou x=1/3.Dessa forma: 
z=5-6=-1 ou z=5-2/3=13/3.Depois só ver que um é mínimo e outro é máximo rs 
Abraços!Claudio Gustavo

Enviado do Yahoo Mail para iPhone


Em quinta-feira, abril 26, 2018, 12:39 AM, cicero calheiros 
<ciceropiano2...@gmail.com> escreveu:

Se x+y+z= 5 e xy+xz+yz=3 . Verifique que -1<=z<=13/3.
Alguém pode ajudar nessa questão
Em Qui, 26 de abr de 2018 00:34, cicero calheiros <ciceropiano2...@gmail.com> 
escreveu:

Se x+y+z=5 e x.y+x.z+y.z =3 . Verifique que  -1=<Z<=13/3
Alguém pode ajudar nessa. Está em um artigo de uma Eureka.


--
Esta mensagem foi verificada pelo sistema de antiv�rus e 
 acredita-se estar livre de perigo.



-- 
Esta mensagem foi verificada pelo sistema de antiv�rus e
 acredita-se estar livre de perigo.



[obm-l] RE: [obm-l] Dúvidas!!

2015-11-26 Por tôpico Gustavo Henrique dos Santos
01. 26 litros de uma solução de álcool + solvente a 30% (ou 30 graus G.L.) 
contêm 26 * 0,30 = 7,8 litros de álcool.Logo, são 26,0 - 7,8 = 18,2 litros de 
solvente.É necessário acrescentar x litros de soluto para que (x + 26) - 0,35 * 
(x + 26) = 18,2, sendo x + 26 o volume finalPortanto, x +26 - 0,35 * x - 9,1 = 
18,2 ==> 0,65 * x + 16,9 = 18,2 ==> 0,65 * x = 1,3 ==> x = 2 litros.Logo, 2 
litros de álcool devem ser adicionados.
02. 90% das crianças estão gripadas. Logo, é provável que 90% * 8% = 7,2% das 
crianças estejam gripadas e tenham manchas vermelhas na pele, e 90% * 92% = 
82,8% das crianças estejam gripadas e NÃO tenham manchas vermelhas na pele.10% 
das crianças estão com rubéola. Logo, é provável que 10% * 95% = 9,5% das 
crianças estejam com rubéola e tenham manchas vermelhas na pele e 10% * 5% = 
0,5% das crianças estejam com rubéola e NÃO tenham manchas vermelhas na pele.Se 
a criança examinada pelo médico tem manchas vermelhas na pele, ela está dentre 
os 7,2% + 9,5% = 16,7% de prováveis crianças com manchas vermelhas na pele. 
Fazendo uma regra de três:
Rubéola: 9,5 - xQualquer doença: 16,7 
--- 100%
percebemos que x = 9,5 * 100% / 16,7. Logo, x é aproximadamente igual a 56,9%, 
que está mais próximo de 57%.Logo, há 57% de chances de que a criança tenha 
rubéola.
From: claudiot...@hotmail.com
To: obm-l@mat.puc-rio.br
Subject: [obm-l] Dúvidas!!
Date: Thu, 26 Nov 2015 02:21:43 +




01.Quantos litros de álcool devem ser adicionados a 26 litros de uma solução 
com 30% de álcool, para obtermos uma segunda solução com concentração de 35% de 
álcool?
02.Um médico foi chamado para examinar uma criança doente. Na vizinhança onde a 
criança mora, 90% das crianças estão gripadas, e os outros 10% estão com 
rubéola. Um sintoma comum de rubéola é o aparecimento de manchas vermelhas na 
pele, o que ocorre com probabilidade de 95%. No caso de gripe, manchas 
vermelhas na pele aparecem com probabilidade de 8%. Se, depois de examinar a 
criança, o médico observa que ela tem manchas vermelhas na pele, qual a 
probabilidade de a criança ter rubéola? Indique o valor inteiro mais próximo do 
valor obtido.
Agradeço Antecipadamente  
--

Esta mensagem foi verificada pelo sistema de antivírus e 

 acredita-se estar livre de perigo.




Este e-mail foi enviado por um computador sem vírus e protegido 
pelo Avast. www.avast.com   


  
-- 
Esta mensagem foi verificada pelo sistema de antivírus e
 acredita-se estar livre de perigo.



Re: [obm-l] Link caronet

2014-04-15 Por tôpico Cláudio Gustavo
Também gostaria do link, por favor.


Em 15/04/2014, às 16:06, Prof Renato Madeira profrenatomade...@gmail.com 
escreveu:

 Poderia me passar o link também. Tenho alguns volumes do Caronet também. Se 
 tiver algum que você não tenha, tento escanear para passá-los aos colegas.
 
 Att, Renato Madeira.
 
 Em 15/04/2014, Ã s 15:48, Igor Battazza batta...@gmail.com escreveu:
 
 Boa tarde Regis,
 
 Gostaria do link.
 
 Abs,
 Igor
 
 
 Em 15 de abril de 2014 15:17, regis barros regisgbar...@yahoo.com.br 
 escreveu:
 Olá Pessoal
 Para aqueles que enviaram e-mail para mim já enviei o link com o 
 caronnet, alguns dos livros estão em francês mas nada que o google 
 tradutor não resolva e outros, ou seja, a grande maioria em português 
 publicado na década de 50. Total de livros 9 volumes.
 Quando o pessoal da lista cita algum livro sempre vou dar uma olhada se 
 tenho no meu acervo, ou compro o livro no estante virtual, caso seja muito 
 raro de se encontrar pego na faculdade e ai faço um scan dele para ter 
 na forma digital ou mesmo uma xerox resolve o problema, mas digo que os 
 livros do caronnet são os mais dificil de se encontrar e realmente vale 
 apena tê-los em casa.
 Algumas criticas é que o pessoal coloca os problemas ou cita eles e 
 não diz da onde retirou o problema e assim fica díficil de ajudar ou 
 mesmo que estamos conversando a mesma lingua.
 Hoje sei que na net tem um bilhão de problemas de olimpiadas de 
 matematica e ai procurar eles e resolve-los é bem divertido, mas tem 
 outras pessoas que não tem tempo para ficar na net garimpando os 
 problemas e assim um site que eu conheço outra pessoa não conhece e 
 assim podemos divulga-los para os demais colegas da lista.
 Não é a primeira vez que o livro do caronnet é citado na lista e 
 alguns anos anteriores já vi esta citação e ai fui na captura dos 
 livros logo é o resultado que alguns de vocês estão recebendo. 
 Logo divulgem o link para os demais colegas.
 
 Uma abração
 
 RegisGBarros 
 
 -- 
 Esta mensagem foi verificada pelo sistema de antivírus e 
 acredita-se estar livre de perigo.
 
 
 -- 
 Esta mensagem foi verificada pelo sistema de antivírus e 
 acredita-se estar livre de perigo.
 
 -- 
 Esta mensagem foi verificada pelo sistema de antivírus e 
 acredita-se estar livre de perigo.

-- 
Esta mensagem foi verificada pelo sistema de antiv�rus e
 acredita-se estar livre de perigo.



Re: [obm-l] n(n+1)(n+2)... (n+p-1) é múltiplo do fatorial de p

2014-04-11 Por tôpico Cláudio Gustavo
Olah!
Bom, sabe-se que, segundo as formulas de combinação e de arranjo:
Cn,p = n!/p!(n-p)!
An,p = n!/(n-p)!
Logo: Cn,p = An,p/p! - An,p = p!Cn,p
Pode-se observar que o produto dado eh: (n+p-1)!/(n-1)! = (n+p-1)!/(n+p-1-p)! = 
A(n+p-1),p
Portanto: A(n+p-1),p = p!C(n+p-1),p
Como o resultado de uma combinação sempre eh inteiro, conclui-se que: 
A(n+p-1),p = p!.k, com k inteiro e dessa forma eh um múltiplo de p! :) 

Enviado via iPhone

Em 11/04/2014, às 19:01, Ennius Lima enn...@bol.com.br escreveu:

 Caros Colegas,
 
 Como podemos provar que o produto n.(n+1).(n+2)... .(n+p-1) é múltiplo do 
 fatorial de p?
 (n e p são naturais maiores do que 1.)
 
 
 Desde já, agradeço-lhes a atenção.
 Abraços do Ennius Lima!
 
 Â 
 
 -- 
 Esta mensagem foi verificada pelo sistema de antivírus e
 acredita-se estar livre de perigo.
 

-- 
Esta mensagem foi verificada pelo sistema de antiv�rus e
 acredita-se estar livre de perigo.


=
Instru��es para entrar na lista, sair da lista e usar a lista em
http://www.mat.puc-rio.br/~obmlistas/obm-l.html
=


Re: [obm-l] Funções

2014-03-10 Por tôpico Cláudio Gustavo
Sejam: f:A-B, g:B-C e a composta h=gof:A-C.

Se h eh injetora queremos provar que f também eh. Sejam a,b elementos de A. 
Fazendo: f(a)=f(b), tem-se que estas imagens sao elementos de B, logo pertencem 
ao dominio de g e podemos aplicar: f(a)=f(b) - g(f(a))=g(f(b)) - h(a)=h(b). 
Pela injetividade de h, tem-se a=b. CQD

Se h eh sobrejetora, queremos provar que g também eh. Supondo por absurdo que g 
não seja sobrejetora, então existe u pertencente ao conjunto C tal que não 
exista nenhum t em B de modo que g(t)=u. Mas como u pertence ao contradomínio 
de h e esta eh sobrejetora, então existe p em A tal que h(p)=u. Logo: 
h(p)=g(f(p))=u. Mas sabe-se que f(p) pertence a B que eh o domínio de g, dessa 
forma existe sim um t=f(p) em B tal que g(t)=u. CQD

Vlw!

Em 10/03/2014, às 08:00, marcone augusto araújo borges 
marconeborge...@hotmail.com escreveu:

 Sejam f e g duas funções f: X -- Y e g: Y-- X.Prove que
 
 a) Se gof é injetiva,então f é injetiva
 
 b) Se fog é sobrejetiva,então g é sobrejetiva
 
 -- 
 Esta mensagem foi verificada pelo sistema de antivírus e 
 acredita-se estar livre de perigo.

-- 
Esta mensagem foi verificada pelo sistema de antiv�rus e
 acredita-se estar livre de perigo.



Re: [obm-l] Polinômio

2014-03-08 Por tôpico Cláudio Gustavo
Observe que (b-a) divide (p(b)-p(a))
Ai que vai gerar o absurdo ;)

Abçs

Em 07/03/2014, às 11:55, marcone augusto araújo borges 
marconeborge...@hotmail.com escreveu:

 Mostre que não existe um polinômio p(x) com coeficientes inteiros tal que
 p(1) = 2,p(2) = 3 e p(3) = 5
 
 -- 
 Esta mensagem foi verificada pelo sistema de antivírus e 
 acredita-se estar livre de perigo.

-- 
Esta mensagem foi verificada pelo sistema de antiv�rus e
 acredita-se estar livre de perigo.



Re: [obm-l] Re: [obm-l] Polinômio

2014-03-08 Por tôpico Cláudio Gustavo
Num polinômio com coeficientes inteiros, ao se substituírem dois valores 
quaisquer a e b do domínio e subtraindo as expressões de p(b) e p(a) eh 
possível colocar o fator b-a em evidencia. Observando que o outro fator que 
multiplica b-a continua sendo inteiro, tem-se que (p(b)-p(a))/(b-a) eh 
inteiro e que b-a divide p(b)-p(a).


Em 08/03/2014, às 10:48, Bernardo Freitas Paulo da Costa 
bernardo...@gmail.com escreveu:

 2014-03-07 12:57 GMT-03:00 Carlos Nehab carlos.ne...@gmail.com:
 Faça p(x) : (x-1)(x-2)(x-3)Q(x) mais ax2 mais bx mais c e aplique as três
 condições.
 Nehab
 
 Isso dá três equações lineares para a, b, c, o que permite
 determiná-los. Eu duvido que eles sejam inteiros, mas eles certamente
 serão racionais. Porque isso seria incompatível com p(x) ter
 coeficientes inteiros ? Não seria possível que Q(x) também tivesse
 coeficientes racionais e com isso cancelasse magicamente os
 racionais que porventura aparecessem em a,b e/ou c ?
 
 Abraços,
 -- 
 Bernardo Freitas Paulo da Costa
 
 -- 
 Esta mensagem foi verificada pelo sistema de antivírus e
 acredita-se estar livre de perigo.
 
 
 =
 Instruções para entrar na lista, sair da lista e usar a lista em
 http://www.mat.puc-rio.br/~obmlistas/obm-l.html
 =

-- 
Esta mensagem foi verificada pelo sistema de antiv�rus e
 acredita-se estar livre de perigo.


=
Instru��es para entrar na lista, sair da lista e usar a lista em
http://www.mat.puc-rio.br/~obmlistas/obm-l.html
=


Re: [obm-l] Re: [obm-l] Re: [obm-l] Polinômio

2014-03-08 Por tôpico Cláudio Gustavo
Ah desculpe! Perfeito ;)

Abçs


Em 08/03/2014, às 16:19, Bernardo Freitas Paulo da Costa 
bernardo...@gmail.com escreveu:

 2014-03-08 14:41 GMT-03:00 Cláudio Gustavo claudiog...@yahoo.com.br:
 Num polinômio com coeficientes inteiros, ao se substituírem dois valores 
 quaisquer a e b do domínio e subtraindo as expressões de p(b) e p(a) eh 
 possível colocar o fator b-a em evidencia. Observando que o outro fator 
 que multiplica b-a continua sendo inteiro, tem-se que (p(b)-p(a))/(b-a) eh 
 inteiro e que b-a divide p(b)-p(a).
 
 Eu não contestei a sua solução, Cláudio. O meu problema é com a
 solução do Nehab. Continuo sem ver como usar a expressão p(x) =
 (x-1)(x-2)(x-3)Q(x) + ax^2 + bx + c ajuda a resolver a questão. A
 divisão euclidiana que ele faz (conforme a outra mensagem dele na
 lista) não garante que Q(x) tem coeficientes inteiros.
 
 Abraços,
 -- 
 Bernardo Freitas Paulo da Costa
 
 -- 
 Esta mensagem foi verificada pelo sistema de antivírus e
 acredita-se estar livre de perigo.
 
 
 =
 Instruções para entrar na lista, sair da lista e usar a lista em
 http://www.mat.puc-rio.br/~obmlistas/obm-l.html
 =

-- 
Esta mensagem foi verificada pelo sistema de antiv�rus e
 acredita-se estar livre de perigo.


=
Instru��es para entrar na lista, sair da lista e usar a lista em
http://www.mat.puc-rio.br/~obmlistas/obm-l.html
=


Re: [obm-l] àlgebra linear

2013-09-03 Por tôpico Cláudio Gustavo
Basta observar que: CA = A^3 + B^2A = B^3 + A^2B = (B^2 + A^2)B = CB
Caso C seja inversível, então: CA = CB, C^-1CA = C^-1CB, IA = IB, A = B.


Em 02/09/2013, às 16:32, douglas.olive...@grupoolimpo.com.br escreveu:

 Se duas matrizes A e B satisfazem Aˆ3=Bˆ3 e (Aˆ2)B=(Bˆ2)A , como mostrar se 
 C=Aˆ2+Bˆ2 é inversivel?
 
 Qualquer ajuda agradeço!
 
 Douglas Oliveira
 
  
 
 -- 
 Esta mensagem foi verificada pelo sistema de antivírus e 
 acredita-se estar livre de perigo.

-- 
Esta mensagem foi verificada pelo sistema de antiv�rus e
 acredita-se estar livre de perigo.



Re: [obm-l] Ajuda

2013-05-12 Por tôpico Cláudio Gustavo
Basta isolar o b e resolver:
b=(4a+9c)/6
Delta=(4a+9c)^2/36 - 4ac = (4a-9c)^2/36
x=(-b+-|4a-9c|/6)/2a = (-6b+-(4a-9c))/12a
Logo:

Enviado via iPhone

Em 12/05/2013, às 12:11, Marcelo de Moura Costa mat.mo...@gmail.com escreveu:

 Determine as raízes da equação aX² + bX + C = 0  sabendo que 4a - 6b + 9c = 0.

=
Instru��es para entrar na lista, sair da lista e usar a lista em
http://www.mat.puc-rio.br/~obmlistas/obm-l.html
=


Re: [obm-l] Ajuda

2013-05-12 Por tôpico Claudio Gustavo
  eh verdade, falta alguma outra condiçao para se determinar o x_1...

--- Em dom, 12/5/13, Eduardo Wilner eduardowil...@yahoo.com.br escreveu:

De: Eduardo Wilner eduardowil...@yahoo.com.br
Assunto: Re: [obm-l] Ajuda
Para: obm-l@mat.puc-rio.br obm-l@mat.puc-rio.br
Data: Domingo, 12 de Maio de 2013, 13:09

Faltam condições...

De: Marcelo de Moura Costa mat.mo...@gmail.com
 Para: obm-l@mat.puc-rio.br 
 Enviadas: Domingo, 12 de Maio de 2013 12:11
 Assunto: [obm-l] Ajuda
   
Determine as raízes da equação aX² + bX + C = 0  sabendo que 4a - 6b + 9c =
 0.



Re: [obm-l] Ajuda

2013-05-12 Por tôpico Claudio Gustavo
perdao o email foi quebrado...Logo:x_1 = -3c/2a e x_2 = -2/3
--- Em dom, 12/5/13, Cláudio Gustavo claudiog...@yahoo.com.br escreveu:

De: Cláudio Gustavo claudiog...@yahoo.com.br
Assunto: Re: [obm-l] Ajuda
Para: obm-l@mat.puc-rio.br obm-l@mat.puc-rio.br
Cc: obm-l@mat.puc-rio.br obm-l@mat.puc-rio.br
Data: Domingo, 12 de Maio de 2013, 14:31

Basta isolar o b e resolver:
b=(4a+9c)/6
Delta=(4a+9c)^2/36 - 4ac = (4a-9c)^2/36
x=(-b+-|4a-9c|/6)/2a = (-6b+-(4a-9c))/12a
Logo:

Enviado via iPhone

Em 12/05/2013, às 12:11, Marcelo de Moura Costa mat.mo...@gmail.com escreveu:

 Determine as raízes da equação aX² + bX + C = 0  sabendo que 4a - 6b + 9c 
 = 0.

=
Instruções para entrar na lista, sair da lista e usar a lista em
http://www.mat.puc-rio.br/~obmlistas/obm-l.html
=


Re: [obm-l] Ajuda

2013-05-12 Por tôpico Claudio Gustavo
Basta isolar o b e resolver:
b=(4a+9c)/6
Delta=(4a+9c)^2/36 - 4ac = (4a-9c)^2/36
x=(-b+-|4a-9c|/6)/2a = (-6b+-(4a-9c))/12a
Logo:

--- Em dom, 12/5/13, Marcelo de Moura Costa mat.mo...@gmail.com escreveu:

De: Marcelo de Moura Costa mat.mo...@gmail.com
Assunto: [obm-l] Ajuda
Para: obm-l@mat.puc-rio.br
Data: Domingo, 12 de Maio de 2013, 12:11

Determine as raízes da equação aX² + bX + C = 0  sabendo que 4a - 6b + 9c = 0.


Re: [obm-l] Ajuda

2013-05-12 Por tôpico Cláudio Gustavo
Aproveitando q o assunto eh eq quadratica, uma questao aparentemente simples q 
acho bem legal eh:
Resolva numericamente a equação (ax-b)^2 + (bx-a)^2 = x. Sendo a e b inteiros e 
a equação possui duas raízes reais distintas.

Enviado via iPhone

Em 12/05/2013, às 23:48, Carlos Yuzo Shine cysh...@yahoo.com escreveu:

 Outra maneira de ver que -2/3 é raiz é notar que 4a - 6b + 9 = 0 é 
 equivalente a a(-2/3)^2 + b(-2/3) + c = 0 (é só dividir tudo por 9). A outra 
 raiz pode ser encontrada com as relações de soma e produto.
 
 []'s
 Shine
 
 
 
 
 
 
 From: Marcelo de Moura Costa mat.mo...@gmail.com
 To: obm-l@mat.puc-rio.br 
 Sent: Sunday, May 12, 2013 12:11 PM
 Subject: [obm-l] Ajuda
 
 
 
 Determine as raízes da equação aX² + bX + C = 0  sabendo que 4a - 6b + 9c = 0.
 
 =
 Instruções para entrar na lista, sair da lista e usar a lista em
 http://www.mat.puc-rio.br/~obmlistas/obm-l.html
 =

=
Instru��es para entrar na lista, sair da lista e usar a lista em
http://www.mat.puc-rio.br/~obmlistas/obm-l.html
=


Re: [obm-l] Eureka 31 - Teorema de Miquel

2013-05-10 Por tôpico Claudio Gustavo
  Boa noite.  Vou passar aqui as etapas mas ajuda se vc, ao ler, tentar refazer 
com lapis e papel.
  A principio,
seja o quadrilátero convexo completo BCED com retas suportes dos lados sendo as
retas que passam pelos pontos BDA, CEA, BCF e DEF (grupos de três pontos
colineares) e seja o ponto M de Miguel. Sendo assim, construímos as 
circunferências
que passam pelos pontos MADE (centro G), MABC (centro J), MECF (centro H) e
MDBF (centro I).

Observamos que
basta demonstrar que o pentágono MGJIH é inscritível que então teremos 
demonstrado
o que se pede. O argumento é demonstrar, separadamente, que os quadriláteros 
MGJH
e MGIH são inscritíveis, pois tendo ambos três pontos em comum necessariamente
estarão na mesma circunferência. 

Inicialmente
vemos que o quadrilátero MADE é inscritível, logo Ang(MAE)=Ang(MDE)=Ang(MGH).
Da mesma forma MABC é inscritível, logo Ang(MAE)=Ang(MAC)=Ang(MBC)=Ang(MJH). 
Sendo
Ang(MGH)=Ang(MJH), temos que o quadrilátero MGJH é inscritível! Procedendo
analogamente conclui-se que o quadrilátero MGIH também é inscritível e logo os
pontos M, G, J, I e H pertencem todos a mesma circunferência. CQD
  AbraçosClaudio Gustavo
--- Em qui, 9/5/13, Martins Rama martin...@pop.com.br escreveu:

De: Martins Rama martin...@pop.com.br
Assunto: [obm-l] Eureka 31 - Teorema de Miquel
Para: obm-l@mat.puc-rio.br
Data: Quinta-feira, 9 de Maio de 2013, 4:56

Caros amigos da lista, o Carlos Yuzo Shine no seu artigo da Eureka 31
propôs a seguinte questão:

Considere um quadrilátero completo. Seja M o seu ponto de Miquel. Prove que:
(a) os circuncentros dos quatro triângulos determinados pelo quadrilátero
e M estão sobre uma mesma circunferência.

Alguma sugestão?

[]'s
Martins Rama.

=
Instruções para entrar na lista, sair da lista e usar a lista em
http://www.mat.puc-rio.br/~obmlistas/obm-l.html
=


Re: [obm-l] Probleminha interessante.

2013-05-08 Por tôpico Cláudio Gustavo
Eh verdade... Eh como se não exercesse a função de tapete!  
Agora entendi o que você quis dizer. Concordo!

Abçs

Enviado via iPhone

Em 07/05/2013, às 23:04, terence thirteen peterdirich...@gmail.com escreveu:

 Entendi. E foi por isso que achei mal formulado. Mas acho que ainda assim dá 
 problema.
 
 Pensa assim: qual a área útil de cada tapete?
 
 É aquela que toca o chão, correto?
 Então, se uma área do tapete tocar outra coisa que não o tapete, ela não é 
 útil. E se uma área do tapete cobrir outras duas, de novo ela é inútil.
 
 Pensa num caso extremo: três tapetes circulares concêntricos.
 
 
 
 Em 7 de maio de 2013 21:59, Cláudio Gustavo claudiog...@yahoo.com.br 
 escreveu:
   Olah!
 Bom talvez eu não tenha sido muito claro na minha explicação, mas não há 
 regiões contadas repetidamente, pois se A sobrepõem B e B a C, a parte de C 
 sob B não conta como área sobreposta de C por A. Somente se houvesse 
 contato entre os tapetes.   
 
 Enviado via iPhone
 
 Em 07/05/2013, às 19:52, terence thirteen peterdirich...@gmail.com 
 escreveu:
 
 
 
 
 Em 6 de maio de 2013 21:37, Cláudio Gustavo claudiog...@yahoo.com.br 
 escreveu:
 Boa noite.
 Imaginei a sobreposição como uma relação apenas entre dois tapetes. Pois 
 vamos imaginar que, por exemplo, os tapetes A, B e C estejam amontoados 
 com uma parte de C embaixo, acima um pedaço de B e acima outro pedaço de 
 A. Teríamos A sobrepondo um pedaço de B, A sobrepondo um outro pedaço 
 diferente de C (sem B no meio) e B sobrepondo o outro pedaço de C.
 
 Mas é de se supor que os tapetes são finos. Assim, um determinado ponto 
 seria coberto por vários e não apenas dois tapetes. Afinal, se pensarmos no 
 quanto de um tapete é utilizado para cobrir área útil, temos que fazer algo 
 assim:
 
 * Contar os tapetes sem levar em conta intersecções
 * Descontar intersecções dois a dois
 * Contar intersecções três a três
 * Descontar intersecções quatro a quatro
 
 E assim por diante. Aí sim, podemos dizer quais áreas estão sendo 
 efetivamente cobertas - sem ocorrer contagens adicionais.
 
 Do jeito que você faz, me parece que os tapetes são como tapetes de verdade 
 e não folhas de papel (como eu pensei). Talvez seja um problema formulado 
 fracamente...
  
 Dessa forma só a área em que X compartilha imediatamente acima de Y é que 
 é contada na sobreposição de X com Y. Assim nenhum pedaço é esquecido e 
 nem contado mais de uma vez.
 
 Na verdade a área comum é contada três vezes em cada par :)
  
 Complementando, considerei o caso extremo em que toda região de área 5 
 seria coberta pelos tapetes, do contrário haveria mais partes sobrepostas.
 
 Abraços
 Claudio Gustavo
 
 Enviado via iPhone
 
 Em 05/05/2013, às 22:58, terence thirteen peterdirich...@gmail.com 
 escreveu:
 
 
 
 
 Em 5 de maio de 2013 17:25, Cláudio Gustavo claudiog...@yahoo.com.br 
 escreveu:
 A principio observa-se que as áreas sobrepostas somam: 9-5=4. Dessa 
 forma, seja, por absurdo, que nao existem dois tapetes com áreas 
 sobrepostas 1/9 ou mais.
 Sendo assim:
 Tapetes que se sobrepõem: k (com k positivo inteiro menor ou igual a 9)
 Tapetes que nao se sobrepõem: 9-k
 Total de formas de se escolher dois tapetes que se sobrepoem: Ck,2 = 
 k(k-1)/2
 Logo: 
 4/(k(k-1)/2)  1/9
 k^2 -k -72  0
 k -8 ou k9 (absurdo)
 
 E se ocorrer tripla sobreposição? Três tapetes dividindo uma área comum? 
 Abraços
 Claudio Gustavo
 
 Enviado via iPhone
 
 Em 05/05/2013, às 16:08, terence thirteen peterdirich...@gmail.com 
 escreveu:
 
 Tentar teoria dos conjuntos, ou inclusão-exclusão?
 
 A soma da área coberta é no máximo 5. 
 Cada um tem tamanho 1
 Nunca há dois que se sobreponham mais de 1/9.
 
 A área coberta deve ser no mínimo o total dos tapetes menos as 
 sobreposições.
 
 São 9 tapetes e 9*4 sobreposições. Total 9-9*4/9=5 sobreposições.
 
 Ixi! Só deu pra provar a igualdade!
 
 
 Em 3 de maio de 2013 14:23, Mauricio de Araujo 
 mauricio.de.ara...@gmail.com escreveu:
 Considere uma sala com área igual a 5. Nesta sala colocamos 9 tapetes 
 de área igual a 1 e com formatos arbitrários. Prove que existem dois 
 tapetes cuja área de sobreposição é maior do que 1/9.
 
 dica: redução ao absurdo. 
 
 -- 
 Abraços
 
 ​M.
 momentos excepcionais pedem ações excepcionais.
 Os cemitérios estão cheios de pessoas insubstituíveis em seus ofícios..
 
 
 
 -- 
 /**/
 神が祝福
 
 Torres
 
 
 
 -- 
 /**/
 神が祝福
 
 Torres
 
 
 
 -- 
 /**/
 神が祝福
 
 Torres
 
 
 
 -- 
 /**/
 神が祝福
 
 Torres


Re: [obm-l] Probleminha interessante.

2013-05-07 Por tôpico Cláudio Gustavo
  Olah!
Bom talvez eu não tenha sido muito claro na minha explicação, mas não há 
regiões contadas repetidamente, pois se A sobrepõem B e B a C, a parte de C sob 
B não conta como área sobreposta de C por A. Somente se houvesse contato 
entre os tapetes.   

Enviado via iPhone

Em 07/05/2013, às 19:52, terence thirteen peterdirich...@gmail.com escreveu:

 
 
 
 Em 6 de maio de 2013 21:37, Cláudio Gustavo claudiog...@yahoo.com.br 
 escreveu:
 Boa noite.
 Imaginei a sobreposição como uma relação apenas entre dois tapetes. Pois 
 vamos imaginar que, por exemplo, os tapetes A, B e C estejam amontoados 
 com uma parte de C embaixo, acima um pedaço de B e acima outro pedaço de A. 
 Teríamos A sobrepondo um pedaço de B, A sobrepondo um outro pedaço diferente 
 de C (sem B no meio) e B sobrepondo o outro pedaço de C.
 
 Mas é de se supor que os tapetes são finos. Assim, um determinado ponto seria 
 coberto por vários e não apenas dois tapetes. Afinal, se pensarmos no quanto 
 de um tapete é utilizado para cobrir área útil, temos que fazer algo assim:
 
 * Contar os tapetes sem levar em conta intersecções
 * Descontar intersecções dois a dois
 * Contar intersecções três a três
 * Descontar intersecções quatro a quatro
 
 E assim por diante. Aí sim, podemos dizer quais áreas estão sendo 
 efetivamente cobertas - sem ocorrer contagens adicionais.
 
 Do jeito que você faz, me parece que os tapetes são como tapetes de verdade e 
 não folhas de papel (como eu pensei). Talvez seja um problema formulado 
 fracamente...
  
 Dessa forma só a área em que X compartilha imediatamente acima de Y é que é 
 contada na sobreposição de X com Y. Assim nenhum pedaço é esquecido e nem 
 contado mais de uma vez.
 
 Na verdade a área comum é contada três vezes em cada par :)
  
 Complementando, considerei o caso extremo em que toda região de área 5 seria 
 coberta pelos tapetes, do contrário haveria mais partes sobrepostas.
 
 Abraços
 Claudio Gustavo
 
 Enviado via iPhone
 
 Em 05/05/2013, às 22:58, terence thirteen peterdirich...@gmail.com 
 escreveu:
 
 
 
 
 Em 5 de maio de 2013 17:25, Cláudio Gustavo claudiog...@yahoo.com.br 
 escreveu:
 A principio observa-se que as áreas sobrepostas somam: 9-5=4. Dessa forma, 
 seja, por absurdo, que nao existem dois tapetes com áreas sobrepostas 1/9 
 ou mais.
 Sendo assim:
 Tapetes que se sobrepõem: k (com k positivo inteiro menor ou igual a 9)
 Tapetes que nao se sobrepõem: 9-k
 Total de formas de se escolher dois tapetes que se sobrepoem: Ck,2 = 
 k(k-1)/2
 Logo: 
 4/(k(k-1)/2)  1/9
 k^2 -k -72  0
 k -8 ou k9 (absurdo)
 
 E se ocorrer tripla sobreposição? Três tapetes dividindo uma área comum? 
 Abraços
 Claudio Gustavo
 
 Enviado via iPhone
 
 Em 05/05/2013, às 16:08, terence thirteen peterdirich...@gmail.com 
 escreveu:
 
 Tentar teoria dos conjuntos, ou inclusão-exclusão?
 
 A soma da área coberta é no máximo 5. 
 Cada um tem tamanho 1
 Nunca há dois que se sobreponham mais de 1/9.
 
 A área coberta deve ser no mínimo o total dos tapetes menos as 
 sobreposições.
 
 São 9 tapetes e 9*4 sobreposições. Total 9-9*4/9=5 sobreposições.
 
 Ixi! Só deu pra provar a igualdade!
 
 
 Em 3 de maio de 2013 14:23, Mauricio de Araujo 
 mauricio.de.ara...@gmail.com escreveu:
 Considere uma sala com área igual a 5. Nesta sala colocamos 9 tapetes de 
 área igual a 1 e com formatos arbitrários. Prove que existem dois 
 tapetes cuja área de sobreposição é maior do que 1/9.
 
 dica: redução ao absurdo. 
 
 -- 
 Abraços
 
 ​M.
 momentos excepcionais pedem ações excepcionais.
 Os cemitérios estão cheios de pessoas insubstituíveis em seus ofícios..
 
 
 
 -- 
 /**/
 神が祝福
 
 Torres
 
 
 
 -- 
 /**/
 神が祝福
 
 Torres
 
 
 
 -- 
 /**/
 神が祝福
 
 Torres


Re: [obm-l] Probleminha interessante.

2013-05-06 Por tôpico Cláudio Gustavo
Boa noite.
Imaginei a sobreposição como uma relação apenas entre dois tapetes. Pois vamos 
imaginar que, por exemplo, os tapetes A, B e C estejam amontoados com uma 
parte de C embaixo, acima um pedaço de B e acima outro pedaço de A. Teríamos A 
sobrepondo um pedaço de B, A sobrepondo um outro pedaço diferente de C (sem B 
no meio) e B sobrepondo o outro pedaço de C. Dessa forma só a área em que X 
compartilha imediatamente acima de Y é que é contada na sobreposição de X com 
Y. Assim nenhum pedaço é esquecido e nem contado mais de uma vez.
Complementando, considerei o caso extremo em que toda região de área 5 seria 
coberta pelos tapetes, do contrário haveria mais partes sobrepostas.

Abraços
Claudio Gustavo

Enviado via iPhone

Em 05/05/2013, às 22:58, terence thirteen peterdirich...@gmail.com escreveu:

 
 
 
 Em 5 de maio de 2013 17:25, Cláudio Gustavo claudiog...@yahoo.com.br 
 escreveu:
 A principio observa-se que as áreas sobrepostas somam: 9-5=4. Dessa forma, 
 seja, por absurdo, que nao existem dois tapetes com áreas sobrepostas 1/9 ou 
 mais.
 Sendo assim:
 Tapetes que se sobrepõem: k (com k positivo inteiro menor ou igual a 9)
 Tapetes que nao se sobrepõem: 9-k
 Total de formas de se escolher dois tapetes que se sobrepoem: Ck,2 = k(k-1)/2
 Logo: 
 4/(k(k-1)/2)  1/9
 k^2 -k -72  0
 k -8 ou k9 (absurdo)
 
 E se ocorrer tripla sobreposição? Três tapetes dividindo uma área comum? 
 Abraços
 Claudio Gustavo
 
 Enviado via iPhone
 
 Em 05/05/2013, às 16:08, terence thirteen peterdirich...@gmail.com 
 escreveu:
 
 Tentar teoria dos conjuntos, ou inclusão-exclusão?
 
 A soma da área coberta é no máximo 5. 
 Cada um tem tamanho 1
 Nunca há dois que se sobreponham mais de 1/9.
 
 A área coberta deve ser no mínimo o total dos tapetes menos as 
 sobreposições.
 
 São 9 tapetes e 9*4 sobreposições. Total 9-9*4/9=5 sobreposições.
 
 Ixi! Só deu pra provar a igualdade!
 
 
 Em 3 de maio de 2013 14:23, Mauricio de Araujo 
 mauricio.de.ara...@gmail.com escreveu:
 Considere uma sala com área igual a 5. Nesta sala colocamos 9 tapetes de 
 área igual a 1 e com formatos arbitrários. Prove que existem dois tapetes 
 cuja área de sobreposição é maior do que 1/9.
 
 dica: redução ao absurdo. 
 
 -- 
 Abraços
 
 ​M.
 momentos excepcionais pedem ações excepcionais.
 Os cemitérios estão cheios de pessoas insubstituíveis em seus ofícios..
 
 
 
 -- 
 /**/
 神が祝福
 
 Torres
 
 
 
 -- 
 /**/
 神が祝福
 
 Torres


Re: [obm-l] Probleminha interessante.

2013-05-05 Por tôpico Cláudio Gustavo
A principio observa-se que as áreas sobrepostas somam: 9-5=4. Dessa forma, 
seja, por absurdo, que nao existem dois tapetes com áreas sobrepostas 1/9 ou 
mais.
Sendo assim:
Tapetes que se sobrepõem: k (com k positivo inteiro menor ou igual a 9)
Tapetes que nao se sobrepõem: 9-k
Total de formas de se escolher dois tapetes que se sobrepoem: Ck,2 = k(k-1)/2
Logo: 
4/(k(k-1)/2)  1/9
k^2 -k -72  0
k -8 ou k9 (absurdo)

Abraços
Claudio Gustavo

Enviado via iPhone

Em 05/05/2013, às 16:08, terence thirteen peterdirich...@gmail.com escreveu:

 Tentar teoria dos conjuntos, ou inclusão-exclusão?
 
 A soma da área coberta é no máximo 5. 
 Cada um tem tamanho 1
 Nunca há dois que se sobreponham mais de 1/9.
 
 A área coberta deve ser no mínimo o total dos tapetes menos as sobreposições.
 
 São 9 tapetes e 9*4 sobreposições. Total 9-9*4/9=5 sobreposições.
 
 Ixi! Só deu pra provar a igualdade!
 
 
 Em 3 de maio de 2013 14:23, Mauricio de Araujo mauricio.de.ara...@gmail.com 
 escreveu:
 Considere uma sala com área igual a 5. Nesta sala colocamos 9 tapetes de 
 área igual a 1 e com formatos arbitrários. Prove que existem dois tapetes 
 cuja área de sobreposição é maior do que 1/9.
 
 dica: redução ao absurdo. 
 
 -- 
 Abraços
 
 ​M.
 momentos excepcionais pedem ações excepcionais.
 Os cemitérios estão cheios de pessoas insubstituíveis em seus ofícios..
 
 
 
 -- 
 /**/
 神が祝福
 
 Torres


Re: [obm-l] ajuda em questão de conjuntos

2013-05-02 Por tôpico Cláudio Gustavo
Devemos usar a desigualdade:
n(AUBUC)=n(A)+n(B)+n(C)
x^2 = 2x-3 + x-2 + 3x-4
x^2 -6x +9 = 0
(x-3)^2 = 0
Logo: x=3.
Sendo, para esse valor, quando ocorre a igualdade, temos que todos os conjuntos 
são disjuntos. Portanto as interseções são todas vazias.

Abraços
Claudio Gustavo 

Enviado via iPhone

Em 02/05/2013, às 20:00, Bruno Rodrigues brunorodrigues@gmail.com 
escreveu:

 Boa noite pessoal!
 To empacado na seguinte questão,e gostaria da ajuda de vocês.Aí vai:
  
 Sejam A, B e C conjuntos tais que n(A) = 2x − 3, n(B) = x − 2,
  n(C) = 3x − 4 e n(A  U B U C ) = x2, onde n(S) é o número de elementos  no 
 conjunto S. Ache n(A ∩ B).
  
 Abraços,
 Bruno


Re: [obm-l] FW: Fibonacci

2013-03-31 Por tôpico Cláudio Gustavo
Devemos usar a igualdade auxiliar: F_m+n+1 = F_m+1F_n+1 + F_mF_n e a igualdade 
na forma mais geral:
F_m+n+k = F_m+1F_n+1F_k+1 + F_mF_nF_k - F_m-1F_n-1F_k-1. Em q o caso pedido 
ocorre qdo m=n=k.
Aplicando indução em k e adotando os casos F_m+n+k e F_m+n+k+1, somando e 
fatorando obteremos: F_m+n+k + F_m+n+k+1 = F_m+1F_n+1(F_k+1 + F_k+2) + 
F_mF_n(F_k + F_k+1) - F_m-1F_n-1(F_k-1 + F_k). Sendo a última igualdade fácil 
de verificar.

Abraços
Claudio Gustavo

Em 30/03/2013, às 10:51, marcone augusto araújo borges 
marconeborge...@hotmail.com escreveu:

 
 
 From: marconeborge...@hotmail.com
 To: obm-l@mat.puc-rio.br
 Subject: Fibonacci
 Date: Fri, 29 Mar 2013 14:04:22 +
 
 Mostre por indução que F_3n = F^3_(n) + F^3_(n+1) - F^3_(n-1)
 
 -- 
 Esta mensagem foi verificada pelo sistema de antivírus e 
 acredita-se estar livre de perigo.

-- 
Esta mensagem foi verificada pelo sistema de antiv�rus e
 acredita-se estar livre de perigo.



Re: [obm-l] RE: [obm-l] Função trigonométrica sem período

2013-03-08 Por tôpico Cláudio Gustavo
Boa noite.
Seja por absurdo o periodo T da funcao  f(x)=cos(x^1/2). Dessa forma, para todo 
x nao negativo, tem-se f(x)=f(x+T). Como vale para todo x nas condicoes acima, 
escolhemos x=0: f(0)=cos0=1. Logo f(0)=f(0+T), o que dah: cos(T^1/2)=1, 
T^1/2=2Qpi sendo Q inteiro. Por outro lado, f(0)=f(0+T)=f(0+2T), logo 
cos((2T)^1/2)=1 e (2T)^1/2=2Hpi, sendo H um inteiro qualquer.
Dividindo os resultados (2T)^1/2 = 2Hpi  por T^1/2 = 2Qpi tem-se 2^1/2 = K, 
sendo K um numero racional qualquer. Dai encontra-se o absurdo logo a funcao 
f(x) dada nao eh periodica.

Espero ter ajudado.
Claudio Gustavo.

Enviado via iPhone

Em 09/03/2013, às 01:15, Artur Costa Steiner steinerar...@gmail.com escreveu:

 Eu vou usar um outro argumento, de caráter geral. Antes, vejamos o seguinte 
 lema:
 
 Seja f de R em R, ou de [0, oo) em R, uma função periódica e não constante. 
 Para todo a  0, a  1,a função g(x) = f(x^a) não é uniformemente contínua.
 
 Prova:
 
 Sabemos que uma função g é uniformemente contínua em seu domínio se, e 
 somente se, para todas sequências (u_n) e (v_n) no domínio de g tais que u_n 
 - v_n  -- 0, tivermos que g(u_n) - g(v_n) -- 0. 
 
 Consideremos inicialmente o caso a  1. Sendo p  0 um período qualquer de f, 
 definamos (u_n) e (v_n) por u_n = (np + u)^(1/a) e v_n =  (np + v)^(1/a), 
 onde u e v são reais tais que f(u)  f(v) (como f não é constante, este 
 números existem). Como a  1, 0   1/a  1, o que implica que u_n - v_n -- 0 
 (isto pode ser deduzido da definição da função potência). Entretanto, para 
 todo n, g(u_n) - g(v_n) = f((u_n)^a) - f((v_n)^a) = f(np + u) - f(np + v) = 
 f(u) - f(v), pois p é período de f. Logo, g(u_n) - g(v_n) converge 
 trivialmente para f(u) - f(v)  0. Isto nos mostra que g não é uniformemente 
 contínua em R.
 
 Na abordagem a seguir, consideramos o fato de que funções contínuas e 
 periódicas são uniformemente contínuas.
 
 Suponhamos agora que, além de periódica e não constante, f seja contínua. 
 Então, f é uniformemente contínua e g é contínua (composição das funções 
 contínuas f e x-- x^a). Se g for periódica, então g, contrariamente ao lema 
 que demonstramos, é uniformemente contínua. Desta contradição, deduzimos que 
 g não é periódica. 
 
 Supondo novamente f contínua, periódica e não constante, consideremos agora o 
 caso a em (0, 1). Então, g não pode ser constante, pois a função não negativa 
 x -- x^a é uma bijeção. Admitamos que g seja periódica. Como 1/a  1, temos 
 do caso anterior que f, dada por f(x) = g(x^(1/a)), não é uniformemente 
 contínua., Uma contradição. Logo, g não é periódica.
 
 Verificamos assim, que, se f for contínua, periódica e não constante, então 
 pata todo a  0, a  1, g não é periódica. Na sua questão, temos o caso 
 particular para f(x) = cos(x) e a = 1/2.
 
 Abraços
 
 Artur Costa Steiner
 
 Em 08/03/2013, às 22:58, Márcio Pinheiro profmar...@hotmail.com escreveu:
 
 Basta resolver a equação cos ((x + t)^1/2) = cos (x^1/2), supondo, 
 inicialmente, que f é periódica, para concluir que, em qualquer solução, t 
 depende de x. Logo, f não pode ser periódica, pois, se fosse, deveria haver 
 t  0, independente de x (ponto de partida), tal que f (x + t) = f (x), para 
 todo x, isto é, não importa qual seja x.
 
 From: marconeborge...@hotmail.com
 To: obm-l@mat.puc-rio.br
 Subject: [obm-l] Função trigonométrica sem período
 Date: Fri, 8 Mar 2013 18:52:48 +
 
 O objetivo dessa questao é demonstrar que f(x) = cos(x^1/2),x  = 0,não é 
 periódica,ou seja,não existe nenhum numero
 real positivo T tal que cos[(x+T)]^1/2 = cos(x^1/2) para todo x  = 0. 
  
 a) Encontre todos os valores de T  = 0 para os quais f(T) = f(0) e,a seguir 
 encontre todos os valores de T  = 0 para os quais
 f(T) = f(2T)
  
 b) use o ítem a para mostrar que f(x) não é periodica 


[obm-l] Função

2010-08-29 Por tôpico Gustavo Souza
Olá a todos, estou com problema na seguinte questão:

Considere a função f: R(^2) - R definida pela expressão:

f( x+y , x-y ) = ( (x^2) - 3*x*y + 2*(y^2) ) / ( (x²) - (y²) )

Exiba f(x,y).

Obrigado


  

Re: [obm-l] RES: [obm-l] Dica de Livro de Matemát ica

2010-07-21 Por tôpico Gustavo Simões Araújo
Pessoal,

   Muito obrigado!! Eu agora estou com várias dicas de livro pra ler...

Abs,

--
Gustavo Simões Araujo

On 20/07/2010, at 18:14, Osmundo Bragança barz...@dglnet.com.br wrote:

 Recomendo - O último teorema de Fermat - autor: Simon Singh - Editora Record
 
 
 Gostei muitíssimo.
 Um abraço.
 Osmundo
 
 -Mensagem original-
 De: owner-ob...@mat.puc-rio.br [mailto:owner-ob...@mat.puc-rio.br] Em nome
 de Gustavo Simões Araújo
 Enviada em: terça-feira, 20 de julho de 2010 13:29
 Para: obm-l@mat.puc-rio.br
 Assunto: [obm-l] Dica de Livro de Matemática
 
 Olá Pessoal, 
 
   Eu estou querendo ler algum livro sobre matemática, podendo ser tanto
 sobre a história da matemática, como sobre algum assunto especifíco, por
 exemplo número inteiros.
 
Eu li o The Music of the Primes (Marcus du Sautoy) e gostei bastante,
 por acaso alguém teria algum outro para indicar? Eu li sobre o Poincaré's
 Prize (George Szpiro) na internet, alguém conhece por acaso? Ou alguém sabe
 algum livro interessante sobre o ultimo Teorema de Fermat?
 
 Abs,
 
 --
 Gustavo Simões Araujo
 =
 Instrues para entrar na lista, sair da lista e usar a lista em
 http://www.mat.puc-rio.br/~obmlistas/obm-l.html
 =
 
 
 =
 Instruções para entrar na lista, sair da lista e usar a lista em
 http://www.mat.puc-rio.br/~obmlistas/obm-l.html
 =

=
Instru��es para entrar na lista, sair da lista e usar a lista em
http://www.mat.puc-rio.br/~obmlistas/obm-l.html
=


[obm-l] Dica de Livro de Matemática

2010-07-20 Por tôpico Gustavo Simões Araújo
Olá Pessoal, 

   Eu estou querendo ler algum livro sobre matemática, podendo ser tanto 
sobre a história da matemática, como sobre algum assunto especifíco, por 
exemplo número inteiros.

Eu li o The Music of the Primes (Marcus du Sautoy) e gostei bastante, por 
acaso alguém teria algum outro para indicar? Eu li sobre o Poincaré's Prize 
(George Szpiro) na internet, alguém conhece por acaso? Ou alguém sabe algum 
livro interessante sobre o ultimo Teorema de Fermat?

Abs,

--
Gustavo Simões Araujo
=
Instru��es para entrar na lista, sair da lista e usar a lista em
http://www.mat.puc-rio.br/~obmlistas/obm-l.html
=


Res: [obm-l] Provando Continuidade

2010-06-23 Por tôpico Gustavo Souza
Qual seria o livro do Elon? Eu pesquisei no banco de livros da biblioteca e na 
internet e não achei nada de calculo...

Vlws e Abraços





De: Pedro Júnior pedromatematic...@gmail.com
Para: obm-l@mat.puc-rio.br
Enviadas: Segunda-feira, 21 de Junho de 2010 3:38:38
Assunto: Re: [obm-l] Provando Continuidade

Vê o livro do Elon vol. 02, lá tem uma série de contra-exemplos como estes que 
o Ralph falou, vê também o APOSTOL talvez o melhor do assunto.
Abraços.


Em 20 de junho de 2010 13:03, Gustavo Souza gustavoandre2006s...@yahoo.com.br 
escreveu:

Nossa, vendo vocês comentarem isso vejo que estou muito fraquinho nessa 
matéria, estou estudando desde o começo do ano pelo livro do Swokowski - 
Calculo com Geometria Analítica, Vol. II e mesmo assim não vejo grandes (nem 
bons) resultados.


Será que alguem teria algum material pra me recomendar (livros, sites, 
apostilas, qlqr coisa)?


Abraços e Obrigado




 De: Ralph Teixeira
 ralp...@gmail.com
Para: obm-l@mat.puc-rio.br
Enviadas: Domingo, 20 de Junho de 2010 11:44:03
Assunto: Re: [obm-l] Provando Continuidade



Eh, do jeito que foi feito, estah errado duas vezes.
 
Primeiro, porque nem todas as retas do plano sao da forma y=tx -- tem a reta 
x=0, que nao pode ser escrita assim.

Segundo, porque ha exemplos classicos de funcoes que sao continuas por todas 
as retas mas nao sao continuas. O que eu tenho no meu bolso eh este:
 
f(x,y) = 2x^2.y/(x^4+y^2) para (x,y)(0,0)
f(0,0)=0
 
Eh facil ver que:
 
i) Se x=0 ou y=0, f eh identicamente nula.
ii) Se y=ax com a0, f(t,at)=2at^3/(t^4+a^2.t^2)=2at/(t^2+a^2) - 0 quando 
t-0.
iii) Porem, f(t,t^2)=1, entao quanto t-0, (t,t^2) se aproxima da origem e 
f(t,t^2)-1.
 
Eh muito bom desenhar as curvas de nivel deste exemplo, o grafico se possivel, 
ver como as retas cortam as curvas de nivel, e entender o que estah 
acontecendo.
 
Abraco, Ralph.

2010/6/20 Bruno França dos Reis bfr...@gmail.com

Do jeito que está na foto, essa resposta está errada.

A afirmação (II) está errada. É errado concluir que o limite de f(x, y) para 
(x, y) tendendo a (0, 0) é 0, usando apenas o que foi afirmado antes.

O que vc provou é que o limite de f(x, y) para (x, y) tendendo a (0, 0) 
através de qualquer reta que passa pela origem vale 0. Mas vc precisa provar 
que f(x, y) tende a (0, 0) quando (x, y) tende a (0, 0) por todo e qualquer 
caminho existente.


Um jeito simples de resolver este exercício é tentar fatorar (x^3 + y^3). 
Dica: (x^3 + y^3) = (x + y)*(...)



--
Bruno FRANÇA DOS REIS

msn: brunoreis...@hotmail.com

skype: brunoreis666
tel: +55 11 9961-7732

http://brunoreis.com
http://brunoreis.com/tech (en)
http://brunoreis.com/blog (pt)

GPG Key: http://brunoreis.com/bruno-public.key

e^(pi*i)+1=0



2010/6/19 Gustavo Souza gustavoandre2006s...@yahoo.com.br 


Ola a todos, sei que esse exercício foge um pouco dos demais aqui da lista, 
mas se alguem puder me dar alguma opinião. Desde ja agradeço.


Enunciado :Verifique a continuidade (a função esta presente no link abaixo) 


http://img257.imageshack.us/img257/807/pergunta.png


Possível Resposta : (No link abaixo)


http://img199.imageshack.us/img199/8342/resposta.png




A minha duvida é: Essa resposta ,do jeito que esta na foto, é considerada 
correta? Se não pq? Se sim pq também?


Obrigado e Abraços
 





   



  

Res: [obm-l] Provando Continuidade

2010-06-20 Por tôpico Gustavo Souza
Nossa, vendo vocês comentarem isso vejo que estou muito fraquinho nessa 
matéria, estou estudando desde o começo do ano pelo livro do Swokowski - 
Calculo com Geometria Analítica, Vol. II e mesmo assim não vejo grandes (nem 
bons) resultados.

Será que alguem teria algum material pra me recomendar (livros, sites, 
apostilas, qlqr coisa)?

Abraços e Obrigado




De: Ralph Teixeira ralp...@gmail.com
Para: obm-l@mat.puc-rio.br
Enviadas: Domingo, 20 de Junho de 2010 11:44:03
Assunto: Re: [obm-l] Provando Continuidade


Eh, do jeito que foi feito, estah errado duas vezes.
 
Primeiro, porque nem todas as retas do plano sao da forma y=tx -- tem a reta 
x=0, que nao pode ser escrita assim.

Segundo, porque ha exemplos classicos de funcoes que sao continuas por todas 
as retas mas nao sao continuas. O que eu tenho no meu bolso eh este:
 
f(x,y) = 2x^2.y/(x^4+y^2) para (x,y)(0,0)
f(0,0)=0
 
Eh facil ver que:
 
i) Se x=0 ou y=0, f eh identicamente nula.
ii) Se y=ax com a0, f(t,at)=2at^3/(t^4+a^2.t^2)=2at/(t^2+a^2) - 0 quando 
t-0.
iii) Porem, f(t,t^2)=1, entao quanto t-0, (t,t^2) se aproxima da origem e 
f(t,t^2)-1.
 
Eh muito bom desenhar as curvas de nivel deste exemplo, o grafico se possivel, 
ver como as retas cortam as curvas de nivel, e entender o que estah acontecendo.
 
Abraco, Ralph.

2010/6/20 Bruno França dos Reis bfr...@gmail.com

Do jeito que está na foto, essa resposta está errada.

A afirmação (II) está errada. É errado concluir que o limite de f(x, y) para 
(x, y) tendendo a (0, 0) é 0, usando apenas o que foi afirmado antes.

O que vc provou é que o limite de f(x, y) para (x, y) tendendo a (0, 0) 
através de qualquer reta que passa pela origem vale 0. Mas vc precisa provar 
que f(x, y) tende a (0, 0) quando (x, y) tende a (0, 0) por todo e qualquer 
caminho existente.


Um jeito simples de resolver este exercício é tentar fatorar (x^3 + y^3). 
Dica: (x^3 + y^3) = (x + y)*(...)



--
Bruno FRANÇA DOS REIS

msn: brunoreis...@hotmail.com
skype: brunoreis666
tel: +55 11 9961-7732

http://brunoreis.com
http://brunoreis.com/tech (en)
http://brunoreis.com/blog (pt)

GPG Key: http://brunoreis.com/bruno-public.key

e^(pi*i)+1=0



2010/6/19 Gustavo Souza gustavoandre2006s...@yahoo.com.br 


Ola a todos, sei que esse exercício foge um pouco dos demais aqui da lista, 
mas se alguem puder me dar alguma opinião. Desde ja agradeço.


Enunciado :Verifique a continuidade (a função esta presente no link abaixo) 


http://img257.imageshack.us/img257/807/pergunta.png


Possível Resposta : (No link abaixo)


http://img199.imageshack.us/img199/8342/resposta.png




A minha duvida é: Essa resposta ,do jeito que esta na foto, é considerada 
correta? Se não pq? Se sim pq também?


Obrigado e Abraços
 




  

[obm-l] Provando Continuidade

2010-06-19 Por tôpico Gustavo Souza
Ola a todos, sei que esse exercício foge um pouco dos demais aqui da lista, mas 
se alguem puder me dar alguma opinião. Desde ja agradeço.

Enunciado :Verifique a continuidade (a função esta presente no link abaixo) 

http://img257.imageshack.us/img257/807/pergunta.png

Possível Resposta : (No link abaixo)

http://img199.imageshack.us/img199/8342/resposta.png


A minha duvida é: Essa resposta ,do jeito que esta na foto, é considerada 
correta? Se não pq? Se sim pq também?

Obrigado e Abraços


  

[obm-l] Analise Combinatória

2009-12-07 Por tôpico Gustavo Duarte
Quem puder ajudar... ,  desde já agradeço.

Uma construtora lançará no 2 º semestre o projeto de 3 edifícios redicencias 
idênticos em uma mesma cidade. para isso, selecionou 6 regiões da cidade com 
perfil   para receber esse tipo de empreendimento. considerando que uma mesma 
região poderá receber, no máximo, dois dos três lançamentos, o número de 
maneiras diferentes de distribuir esses lançamentos entre as seis regiões é 
igual a:

Encontrei como solução 50, fiz pelo modo PONTO/BARRA  , a+b+c+d+e+f=3, 
pemutação  com repetição de 8 elementos ,em que um repete 3 vz e outro 5 vz, 
isso é igual 56 menos os 6 casos em que cada região recebe os 3 lançamentos 
logo: 56 -6 = 50, no entanto recebi como gabarito 60 !! se errei onde errei ?? 
mais uma vez obrigado por qualquer ajuda.

[obm-l] Lucro Máximo

2009-11-09 Por tôpico Gustavo Duarte
É uma questão básica de ponto máximo/ mínimo,  mas nem tanto 


 Ao preço de $ 100,00 cada exemplar, um vendedor ambulante vende 180 
exemplares de um uma mercadoria com um custo unitário de $ 40,00 o exemplar. 
Este vendedor estima que, para cada $ 5,00 de desconto no preço unitário, fará 
aumentar 30 exemplares nas vendas. O preço da mercadotia para maximizar O LUCRO 
desse vendeor é :  

Dúvida : na função da venda :V (X) = (100 - 5x).( 180 + 30x) , Xv = 7  logo 
preço = $ 65

   na função do lucro : L (X)  = ( 60 - 5x) . ( 180 + 30x) , Xv = 3 , 
logo lucro = 45, assim prço ( venda) = 45 + 40 = $85.

Em qual preço (de venda) terei um lucro máximo ???  Quem puder ajudar ,desde já 
agredeço .

[obm-l] O déficit de meninas na matemática (competitiva, acadêmica)

2009-11-05 Por tôpico gustavo

Olá pessoal da lista, essa é minha primeira postagem aqui!

Me chamou a atenção a publicação de um artigo que discute a disparidade 
entre os gêneros na matemática
a partir das experiências de um professor do departamento de economia do 
MIT com a AMC, a American Mathematics Competition;
também mostrando dados da IMO e a Olimpíada de Matemática Americana 
(USAMO) e do SAT, entre outros.


---
O artigo:

http://econ-www.mit.edu/files/4298

O 'press release':

http://web.mit.edu/newsoffice/2009/math-gender.html
---

Mais do que a diferença de gêneros, esse artigo mostra uma fator 
importante: como o ambiente e os estímulos

influenciam no desempenho em matemática (leia o artigo para entender).


Abraço,
Gustavo Ramires




=
Instruções para entrar na lista, sair da lista e usar a lista em
http://www.mat.puc-rio.br/~obmlistas/obm-l.html
=


[obm-l] O déficit de meninas na matemática

2009-11-04 Por tôpico gustavo

Olá pessoal da lista, esse é minha primeira postagem aqui!

Me chamou a atenção a publicação de um artigo que discute a disparidade 
entre gêneros
a partir das experiências de um professor do departamento de economia do 
MIT com a AMC, a American Mathematics Competition;
também mostrando dados da IMO e a Olimpíada de Matemática Americana 
(USAMO) e do SAT.


---
O artigo:

http://econ-www.mit.edu/files/4298

O 'press release':

http://web.mit.edu/newsoffice/2009/math-gender.html
---

Mais do que a diferença de gêneros, esse artigo mostra uma fator 
importante: como o ambiente e os estímulos

influenciam no desempenho em matemática (leia o artigo para entender).


Gustavo Ramires



=
Instruções para entrar na lista, sair da lista e usar a lista em
http://www.mat.puc-rio.br/~obmlistas/obm-l.html
=


[obm-l] Trigonometria ( equação tg)

2009-10-08 Por tôpico Gustavo Duarte
A questão apresenta  a seguinte equação: tg X = tg ( 9pi )/4 , pergunta-se:

 1) A solução X =  ( 9pi)/4 + Kpi  dada como gabarito desta equação  é 
iquivalente a soluão  X = pi/4 + Kpi  ( com k inteiro)?

2) A equação dada é equivalente a tg x = tg pi/4  ou seja possui soluções 
iguais ?

Desde já agradeço alguma ajuda !!



Re: [obm-l] Trigonometria ( equação tg)

2009-10-08 Por tôpico gustavo

Gustavo Duarte wrote:
A questão apresenta  a seguinte equação: *tg X = tg ( 9pi )/4 *, 
pergunta-se:
 
 1) A solução *X =  ( 9pi)/4 + Kpi * dada como gabarito desta 
equação  é iquivalente a soluão  *X = pi/4 + Kpi*  ( com k inteiro)?
 
2) A equação dada é equivalente a *tg x = tg pi/4  *ou seja possui 
soluções iguais ?
** 
Desde já agradeço alguma ajuda !!
** 
 

Primeiro post na lista ;)

1- Sim. Veja:

9pi/4 + Kpi = pi/4 + (K+2)pi
pi/4 + (K+2)pi = pi/4 + K'pi

Lembrando que tg x = tg ( x +Kpi)

2-Sim. Temos tg x = tg (9pi/4 + Kpi)
como mostrei antes,

9pi/4 + Kpi = pi/4 + (K+2)pi
pi/4 + (K+2)pi = pi/4 + K'pi
K'=K-2 (V) = K inteiro

Abraços,
Gustavo Ramires

=
Instruções para entrar na lista, sair da lista e usar a lista em
http://www.mat.puc-rio.br/~obmlistas/obm-l.html
=


[obm-l] Trigonometria

2009-06-16 Por tôpico Gustavo Simoes Araujo
Pessoal,

 Por acaso alguém sabe como poderia provar que existe um x tal que a
inequação abaixo é verdadeira, sendo w1/w2 irracional e b um numero real
menor que 4:

cos(w1*x) + cos(w2*x) = (b-4)/4

Abraços,

-- 
Gustavo Simões Araújo


Re: [obm-l] Ajuda em um problema

2009-04-10 Por tôpico Gustavo Duarte
Comungo da sua opinião , acho tb que falta alguma informação.
  - Original .Message - 
  From: marcone augusto araújo borges 
  To: obm-l@mat.puc-rio.br 
  Sent: Friday, April 10, 2009 7:48 AM
  Subject: RE: [obm-l] Ajuda em um problema


  Antonio,acho q faltou uma informação sobre a quantidade de alunos reprovados 
tanto em matemática como em portugues.Por exemplo:´´o número de reprovados nas 
duas é a metade do numero de aprovados´´.Do jeito q está o enunciado,poderiamos 
ter tambem 43 aprovados,129 em matematica(3*43),172 em portugues(4*43) e 2 
reprovados nas duas.Total:86(aprov.só em mat)+129(aprov só em port)+43(aprov 
nas duas)+2(reprov nas duas)=260.Se eu estiver errado,q alguem corrija.Um 
abraço. 
   

--
  Date: Thu, 9 Apr 2009 14:39:23 -0300
  Subject: [obm-l] Ajuda em um problema
  From: antoniomcdel...@gmail.com
  To: obm-l@mat.puc-rio.br

  Por favor, necessito ajuda no seguinte problema

  Para passar em um concurso, o aluno deve ser aprovado nas provas de Português 
e Matemática. O número de alunos aprovados em Português é o quádruplo do número 
de aprovados, e o número de aprovados em Matemática é o triplo do número de 
aprovados. O número total de alunos é 260. Quantos foram reprovados?
  Resposta: 220 alunos reprovados.  

  Grato, Antonio del Rio




--
  Quer saber qual produto Windows Live combina melhor com o seu perfil? Clique 
aqui e descubra!

  __ NOD32 3994 (20090407) Information __

  This message was checked by NOD32 antivirus system.
  http://www.eset.com


[obm-l] Re: [obm-l] Re: [obm-l] Conceito de funçao ??

2009-04-05 Por tôpico Gustavo Duarte
Valeu Erick e Valeu Bruno !!! OBG., vejam se entendi certo  !!!
Então é assim, se em um determinado intervalo do domínio a função apresnta 
trechos crescente e trechos constantes ela é CRESCENTE , porém se for crescente 
de ponta a ponta nesse intervalo chamamos ESTRITAMENTE CRESCENTE. ou ainda 
podia ser assim se a função não tiver nenhum pedaço decrescente no intervalo 
estudado,ela é crescente , mesmo que tenha um pedaço constante. AÍ eu 
pergunto  :se nesse intervalo ela for constante de ponta a ponta poderia chamar 
de Crecente  ou decrescente  ?? ou nesse caso seria constante , para esse 
intervalo ??  Abraços , desde já agredeço pela ajuda !!
 o segundo e o terceiro questionamento ficaram OK !!
  - Original Message - 
  From: Erick Nascimento 
  To: obm-l@mat.puc-rio.br 
  Sent: Friday, April 03, 2009 7:47 PM
  Subject: [obm-l] Re: [obm-l] Conceito de funçao ??


  1)


  f é crescente se, e somente se,
  Para todo x e x' (x'  x) no domínio de f, f(x') = f(x).


  f é estritamente crescente se, e somente se,
  Para todo x e x' (x'  x) no domínio de f, f(x')  f(x).



  2)


  f é monótona se uma das afirmações abaixo é verdadeira para f:
  f é estritamente crescente;
  f é estritamente decrescente;
  f é crescente;
  f é decrescente;


  3)


  Você pode ler a frase da seguinte maneira:

  toda função de R em R é a soma de uma Par com outra Impar, pois f(x) = [ 
f(x) + f(-x) ] /2   + [ f(x) - f(-x) ] /2



  A frase diz que toda função f(x) pode ser escrita como:

  f(x) = ( f(x) + f(-x) )/2 + ( f(x) - f(-x) )/2

  Vemos que a primeira função da soma, h(x) = ( f(x) + f(-x) )/2 é par, pois 
h(-x) = ( f(-x) + f(x) )/2 = ( f(x) + f(-x) )/2 = h(x).
  A segunda função da soma, g(x) = ( f(x) - f(-x) )/2 é ímpar, pois g(-x) = ( 
f(-x) - f(x) )/2 = - ( f(x) - f(-x) )/2 = -g(x).


  Portanto, f(x) = h(x) + g(x), onde h é uma função par e g é uma função ímpar.


  Ajudei?


  --
  Erick Nogueira do Nascimento
  Engenheiro de Computação - Unicamp
  Mestrando em Ciência da Computação - IC - Unicamp


  2009/4/3 Gustavo Duarte gvdua...@hotlink.com.br

1) Qual a diferença entre uma função crescente e uma função estritamente 
crescente ?


2) O que seria uma funçõa monótona ( ou monotónica ) ?


3)  Conheço a definiçõa de  função PAR e de IMPAR ,porém não estou 
conseguindo associar a frase com a expressão matematica ; toda função de R em 
R é a soma de uma Par com outra Impar, pois f(x) = [ f(x) + f(-x) ] /2   + [ 
f(x) - f(-x) ] /2Alguém ajuda ?




[obm-l] Conceito de funçao ??

2009-04-03 Por tôpico Gustavo Duarte
1) Qual a diferença entre uma função crescente e uma função estritamente 
crescente ?


2) O que seria uma funçõa monótona ( ou monotónica ) ?


3)  Conheço a definiçõa de  função PAR e de IMPAR ,porém não estou conseguindo 
associar a frase com a expressão matematica ; toda função de R em R é a soma 
de uma Par com outra Impar, pois f(x) = [ f(x) + f(-x) ] /2   + [ f(x) - f(-x) 
] /2Alguém ajuda ?

[obm-l] Re: Lema de Cesàro

2009-03-29 Por tôpico Gustavo Simoes Araujo
Relançando..^_^


2009/3/27 Gustavo Simoes Araujo gustavo.simo...@gmail.com

 Pessoal,

Estou tendo dificuldade com um outro exercicio do lema de
 Cesàro. Sera que alguém poderia me ajudar ? Como eu faço pra provar que:

 sn = \sum_ {k=-n}^n \exp_ (ikx)/2

 \lim_ {n \rightarrow \infty} 1/(n+1) \sum_ {j=0}^n sj =0


 Abraços,

 --
 Gustavo Simões Araújo

 Ps: Se o pessoal preferir visualiza em Latex segue um link :  \lim_ {n
 \rightarrow \infty} 1/(n+1) \sum_ {j=0}^n sj =0




-- 
Gustavo Simões Araújo


[obm-l] Re: Lema de Cesàro

2009-03-29 Por tôpico Gustavo Simoes Araujo
Relançando

2009/3/27 Gustavo Simoes Araujo gustavo.simo...@gmail.com

 Bom dia Pessoal,

 Eu estou precisando de uma ajuda de vocês. Eu queria provar o
 lema de Cesàro pra uma sequência tj, isto é queria provar que se temos:

 qn = 1/(n+1)\sum_ {j=0}^n tj

 \lim_ {n \rightarrow \infty} qn = T

 onde T= \lim_ {n \rightarrow \infty} tj

Para isso eu estou mostrando que apesar de qn, no caso de n par,
 ser diferente de qn, quando n é impar, ambos tende pra T quando n vai pra
 infinito. Esse raciocinio é suficiente pra mostrar que o limite de qn tende
 pra infinito ?

 Abraços,

 --
 Gustavo Simões Araújo




-- 
Gustavo Simões Araújo


[obm-l] Lema de Cesàro

2009-03-27 Por tôpico Gustavo Simoes Araujo
Bom dia Pessoal,

Eu estou precisando de uma ajuda de vocês. Eu queria provar o
lema de Cesàro pra uma sequência tj, isto é queria provar que se temos:

qn = 1/(n+1)\sum_ {j=0}^n tj

\lim_ {n \rightarrow \infty} qn = T

onde T= \lim_ {n \rightarrow \infty} tj

   Para isso eu estou mostrando que apesar de qn, no caso de n par,
ser diferente de qn, quando n é impar, ambos tende pra T quando n vai pra
infinito. Esse raciocinio é suficiente pra mostrar que o limite de qn tende
pra infinito ?

Abraços,

-- 
Gustavo Simões Araújo


[obm-l] Re: Lema de Cesàro

2009-03-27 Por tôpico Gustavo Simoes Araujo
Pessoal,

   Estou tendo dificuldade com um outro exercicio do lema de Cesàro.
Sera que alguém poderia me ajudar ? Como eu faço pra provar que:

sn = \sum_ {k=-n}^n \exp_ (ikx)/2

\lim_ {n \rightarrow \infty} 1/(n+1) \sum_ {j=0}^n sj =0


Abraços,

-- 
Gustavo Simões Araújo

Ps: Se o pessoal preferir visualiza em Latex segue um link :  \lim_ {n
\rightarrow \infty} 1/(n+1) \sum_ {j=0}^n sj =0


[obm-l] Re: [obm-l] Questão CHATA ???

2008-12-17 Por tôpico Gustavo Duarte
Eu tb interpretei esse problema ,como tendo 4 soluções, ou seja dos 5 fz, cada 
um pega 1, restando 3 para distribuir entre os 2 soldados, então x + y = 3, com 
x e y naturais quasquer, temos 4 soluções.( 0,3) (1,2) (2,1) e (3,0).
  - Original Message - 
  From: Fabio Henrique 
  To: obm-l@mat.puc-rio.br 
  Sent: Wednesday, December 17, 2008 8:39 PM
  Subject: [obm-l] Questão CHATA ???


  Essa questão é de um concurso que eu fiz e eu nao consigo entender o gabarito 
, espero que me ajudem , sem mais Fábio

  Dois soldados serão designados para uma mesma missão 
  e  para  eles  serão  distribuídos  (sem  sobra)  5  fuzis  de  tal 
  forma  que  cada  soldado  receba  ao menos  um  fuzil. Essa 
  distribuição deverá ser feita de n formas.  
  Então, pode-se afirmar que n vale : 

  Não vou postar a reposta pois pode interferir na resolução do problema. Desde 
já Obrigado.

  -- 
  Be Free 
  Use LINUX
  Linux #244712


Re: [obm-l] ALGARISMO 1

2008-11-23 Por tôpico Gustavo Duarte
PARABÉNS ,Carlos Alberto !  Saiba que tinha feito alguns tentativas para 
simplificar a  resolução dessa questão porém nenhuma tão legal feito essa , sou 
fã das resoluções desse tipo, claro sem desmerecer todas as outras . Valeu a 
TODOS que ajudaram nessa questaõ.
 
  - Original Message - 
  From: Carlos Alberto da Silva Victor 
  To: obm-l@mat.puc-rio.br 
  Sent: Saturday, November 22, 2008 10:45 PM
  Subject: Re: [obm-l] ALGARISMO 1


  Olá  Arkon ,
  Se  você  posicionar o dígito  1 como último  algarismo , podemos  colocar 
de 0  até 111 nas  outras posições ; ou  seja  112 números . Observe que  o 
mesmo fato  ocorrerá  quando  posicionar o  1 com  algarismo  das  dezenas ; 
ou seja 112 números. Usando   o mesmo argumento   para  as outras  posições  do 
dígito  1, teremos  um total  de  448 vezes  em que  o algarismo  1 será  
escrito , ok ?
  Obs : quando ,por exemplo , o 1 estiver posicionado como dígito das 
centenas , o  zero ( que  corresponde  um  dos 112 números : de  0  até  111) 
que  será  colocado nas  outras  posições  corresponde  ao número 0100 , ok ?
  Abraços

  Carlos  Victor  

   
  Em 22/11/08, arkon [EMAIL PROTECTED] escreveu: 
Pessoal essa é muito trabalhosa, alguém pode resolver de um modo simples, 
por favor

Escrevendo-se todos os números inteiros de 1 a , quantas vezes o 
algarismo 1 é escrito?

A) 289. B) 300.   C) 420.  D) 448.   E) 481.

Gabarito: D) 448.

= 
Instru�ões para entrar na lista, sair da lista e usar a lista em 
http://www.mat.puc-rio.br/~obmlistas/obm-l.html 
= 



[obm-l] Re: [obm-l] Re: [obm-l] PA ( literal ) e aritm ética.

2008-11-20 Por tôpico Gustavo Duarte
OBRIGADO PAULO !!  agora ficou mais do que claro, veleu pelo  passo a passo !!!
  - Original Message - 
  From: Paulo André 
  To: obm-l@mat.puc-rio.br 
  Sent: Thursday, November 20, 2008 12:13 AM
  Subject: Re: [obm-l] Re: [obm-l] PA ( literal ) e aritmética.


  a0 é o primeiro termo da PA e r a razão
  A partir dele podemos descobrir os outros, assim nossa soma fica
  a0 + (a0 + r) + (a0 + 2*r) + ... + (a0 + (m-1)*r) = N
  Repare que a nossa PA tem m termos pois vai de 0 até m-1
  Assim a soma será 
  a0*(m) + r*(1 + 2 + ... + (m-1)) = N
  Para calcular a soma 1 + 2 + ... + m-1=m*(m-1)/2 podemos fazer de muitos 
jeitos. Pode ser feito por indução finita mas tem um jeito mais simples que é 
somar do seguinte modo:
  1+ (m-1) + 2 + (m-2) + 3 + (m-3) + ... = m + m + m + ... = m*(m-1)/2
  Assim chegamos naquela formula.

  Qualquer duvida pode perguntar de novo

  Paulo André


  2008/11/19 Gustavo Duarte [EMAIL PROTECTED]

Paulo obrigado pela ajuda, porém , desculpa, eu entendi todo o seu 
densolvimento, exceto as primeiras equações :
a0*m+m*(m-1)*r/2=N , quem é a0*m ? e porque m*(m-1) ? desde já agradeço .


:- Original Message - 
  From: Paulo André 
  To: obm-l@mat.puc-rio.br 
  Sent: Wednesday, November 19, 2008 11:14 AM
  Subject: Re: [obm-l] PA ( literal ) e aritmética.


  O primeiro problema também não é nenhum bicho de sete cabeças.

  Aplique a fórmula da soma da PA:
  a0*m+m*(m-1)*r/2=N = a0+ r * (m-1)/2=N/m
  a0*N+N*(N-1)*r/2=m = a0 + r * (N-1)/2=m/N
  Subtraia as duas equações
  r(m - N)/2=N/m - m/N=(N^2-m^2)/Nm = (N-m)(N+m)/Nm
  Cortando (N-m)

  r = - 2 (N+m)/N*m

  Paulo André



  2008/11/19 Ralph Teixeira [EMAIL PROTECTED]

  2) Se, x ,y e z são inteiros positivos , com : xyz + xy + xz + yz + x 
+ y + z = 384, quanto vale xyz ?
 GAB. 240


Some um dos dois lados e fatore tudo:

(x+1)(y+1)(z+1)=385=5.7.11

Como x,y e z sao inteiros positivos, x+1,y+1,z+1=2. Como aquela ali eh 
a fatoracao de 385 em primos, a unica opcao eh que {x+1,y+1,z+1}={5,7,11}, isto 
eh, x, y e z sao 4, 6 e 10 em alguma ordem. Assim, xyz=240.

Abraco,
   Ralph





[obm-l] Re: [obm-l] PA ( literal ) e aritmética.

2008-11-19 Por tôpico Gustavo Duarte
Paulo obrigado pela ajuda, porém , desculpa, eu entendi todo o seu 
densolvimento, exceto as primeiras equações :
a0*m+m*(m-1)*r/2=N , quem é a0*m ? e porque m*(m-1) ? desde já agradeço .


:- Original Message - 
  From: Paulo André 
  To: obm-l@mat.puc-rio.br 
  Sent: Wednesday, November 19, 2008 11:14 AM
  Subject: Re: [obm-l] PA ( literal ) e aritmética.


  O primeiro problema também não é nenhum bicho de sete cabeças.

  Aplique a fórmula da soma da PA:
  a0*m+m*(m-1)*r/2=N = a0+ r * (m-1)/2=N/m
  a0*N+N*(N-1)*r/2=m = a0 + r * (N-1)/2=m/N
  Subtraia as duas equações
  r(m - N)/2=N/m - m/N=(N^2-m^2)/Nm = (N-m)(N+m)/Nm
  Cortando (N-m)

  r = - 2 (N+m)/N*m

  Paulo André



  2008/11/19 Ralph Teixeira [EMAIL PROTECTED]

  2) Se, x ,y e z são inteiros positivos , com : xyz + xy + xz + yz + x + y 
+ z = 384, quanto vale xyz ?
 GAB. 240


Some um dos dois lados e fatore tudo:

(x+1)(y+1)(z+1)=385=5.7.11

Como x,y e z sao inteiros positivos, x+1,y+1,z+1=2. Como aquela ali eh a 
fatoracao de 385 em primos, a unica opcao eh que {x+1,y+1,z+1}={5,7,11}, isto 
eh, x, y e z sao 4, 6 e 10 em alguma ordem. Assim, xyz=240.

Abraco,
   Ralph



[obm-l] PA ( literal ) e aritmética.

2008-11-18 Por tôpico Gustavo Duarte

Se alguém puder ajudar em alguma delas ...,desde já agradeço.

 1) Se a soma dos N primeiros termos de uma PA é m e soma dos m primeiros 
termos é N, com N diferente de m, qual a razão da PA ?
   GAB. 2( m + N) / (m.N)


2) Se, x ,y e z são inteiros positivos , com : xyz + xy + xz + yz + x + y + z = 
384, quanto vale xyz ?
   GAB. 240

[obm-l] Re: [obm-l] Re: [obm-l] PA com Função do 1 º grau.

2008-10-28 Por tôpico Gustavo Duarte
João, desculpe eu não ter entendido,para mim ainda não ficou claro... essa PA 
tem 20 termos é isso ?, qual seria o primeiro e qual seraia o último, usando na 
fórmula dá certo ?? onde estou errando ? pensei assim...se a1 = 4,4  , a razão 
é 0,1 e an = 6,4 , n = ?? não seria 21 ? porém por outro lado vc tem razão ,do 
início de 1980 ate o final de 1999 realmente são 20 anos, O GABARITO TÀ CERTO È 
108 MESMO ??  desde ja agradeço pela atenção !!
  - Original Message - 
  From: João Luís 
  To: obm-l@mat.puc-rio.br 
  Sent: Tuesday, October 28, 2008 7:30 AM
  Subject: [obm-l] Re: [obm-l] PA com Função do 1 º grau.


  De 1980 a 1999, são realmente 20 termos: 1999 - 1979 = 20.

  E porque  - 1979? Porque, como o 1980 está incluido nas condições do 
problema, não deve ser subtraído.

  Um abraço a todos,

  João Luís.


- Original Message - 
From: Gustavo Duarte 
To: Olimpíada 
Sent: Monday, October 27, 2008 10:59 PM
Subject: [obm-l] PA com Função do 1 º grau.


O número em trilhões de cigarros vendiddos, anualmente, é dado pela função 
C(t) = 0,1t + 4,4  , com t=0 correspondendo a 1980 ( contado a partir do início 
de 1980). Qual o número em trilhões de cigarros vendidos desde o início de 1980 
até o final de 1999 ?

a) 109   b) 108   c) 107   d) 106  e) 105

O Gabarito dado é B) 108, fiz  e cheguei ao resultado assim , soma de PA,  
[ ( 4,4 + 6,4)20 ] / 2 = 108, porém depois fazendo uma segunda análise 
discordei , pois essa PA não tem 20 termos e sim 21, porém o período dado é de 
20 anos, e aí ... alguém pode me ajudar na interpretação dessa questão ??? 
desde já agradeço.


  __ NOD32 3562 (20081028) Information __

  This message was checked by NOD32 antivirus system.
  http://www.eset.com


[obm-l] Re: [obm-l] Re: [obm-l] Re: [obm-l] Re: [obm-l] PA com Função do 1 º grau.

2008-10-28 Por tôpico Gustavo Duarte
Valeu  João !!! pensei assim: 1980 = C (0) =4,4 ;  1981 = C(1) =4,5 : 1982 = C 
(2) = 4,6 ;  . ;  1999 = c (19) = 6,3 ; 2000 = C ( 20)  = 6,4. fiz até 
2000, pois ele fala no final de 1999. favor observar no final deste e-mail a 
fórmula que usei. Até mais
  - Original Message - 
  From: João Luís 
  To: obm-l@mat.puc-rio.br 
  Sent: Tuesday, October 28, 2008 1:16 PM
  Subject: [obm-l] Re: [obm-l] Re: [obm-l] Re: [obm-l] PA com Função do 1 º 
grau.


  Veja Gustavo, na verdade eu não resolvi o problema, apenas disse que essa PA 
deve ter 20 termos, pois são 20 anos, concorda? 1980 (inclusive) a 1999 
(inclusive).

  De onde você tirou que an = 6,4? Não entendi. DEsculpe-me se estou deixando 
passar algo óbvio, pois estou analisando as coisas meio na pressa aqui.

  Mais tarde, olho pro problema com toda a calma do mundo e posto uma mensagem 
aqui, ok?

  Um abraço a todos,

  João Luís.
- Original Message - 
From: Gustavo Duarte 
To: obm-l@mat.puc-rio.br 
Sent: Tuesday, October 28, 2008 10:11 AM
Subject: [obm-l] Re: [obm-l] Re: [obm-l] PA com Função do 1 º grau.


João, desculpe eu não ter entendido,para mim ainda não ficou claro... essa 
PA tem 20 termos é isso ?, qual seria o primeiro e qual seraia o último, usando 
na fórmula dá certo ?? onde estou errando ? pensei assim...se a1 = 4,4  , a 
razão é 0,1 e an = 6,4 , n = ?? não seria 21 ? porém por outro lado vc tem 
razão ,do início de 1980 ate o final de 1999 realmente são 20 anos, O GABARITO 
TÀ CERTO È 108 MESMO ??  desde ja agradeço pela atenção !!
  - Original Message - 
  From: João Luís 
  To: obm-l@mat.puc-rio.br 
  Sent: Tuesday, October 28, 2008 7:30 AM
  Subject: [obm-l] Re: [obm-l] PA com Função do 1 º grau.


  De 1980 a 1999, são realmente 20 termos: 1999 - 1979 = 20.

  E porque  - 1979? Porque, como o 1980 está incluido nas condições do 
problema, não deve ser subtraído.

  Um abraço a todos,

  João Luís.


- Original Message - 
From: Gustavo Duarte 
To: Olimpíada 
Sent: Monday, October 27, 2008 10:59 PM
Subject: [obm-l] PA com Função do 1 º grau.


O número em trilhões de cigarros vendiddos, anualmente, é dado pela 
função C(t) = 0,1t + 4,4  , com t=0 correspondendo a 1980 ( contado a partir do 
início de 1980). Qual o número em trilhões de cigarros vendidos desde o início 
de 1980 até o final de 1999 ?

a) 109   b) 108   c) 107   d) 106  e) 105

O Gabarito dado é B) 108, fiz  e cheguei ao resultado assim , soma de 
PA,  [ ( 4,4 + 6,4)20 ] / 2 = 108, porém depois fazendo uma segunda análise 
discordei , pois essa PA não tem 20 termos e sim 21, porém o período dado é de 
20 anos, e aí ... alguém pode me ajudar na interpretação dessa questão ??? 
desde já agradeço.


  __ NOD32 3562 (20081028) Information __

  This message was checked by NOD32 antivirus system.
  http://www.eset.com



  __ NOD32 3562 (20081028) Information __

  This message was checked by NOD32 antivirus system.
  http://www.eset.com


[obm-l] Re: [obm-l] Re: [obm-l] PA com Função d o 1 º grau.

2008-10-28 Por tôpico Gustavo Duarte
Também tenho sua opinião, o gabarito deveria ser 107 trilhões, vou verificar a 
fonte caso tenha informação de mudança de gabarito , te mando um e-mail, obg 
pela atenção.
  - Original Message - 
  From: João Luís 
  To: obm-l@mat.puc-rio.br 
  Sent: Tuesday, October 28, 2008 7:25 PM
  Subject: [obm-l] Re: [obm-l] PA com Função do 1 º grau.


  Isso. Mas o 2000 não conta, né? é do início de 1980 até o final de 1999, 
então o ano 2000 tá fora.

  Na minha opinião, esse problema deve ser resolvido assim:
  C(0) = 0,1*0 + 4,4 (trilhões de cigarros vendidos em 1980)
  C(1) = 0,1*1 + 4,4 (trilhões de cigarros vendidos em 1981)
  C(2) = 0,1*2 + 4,4 (trilhões de cigarros vendidos em 1982)
  .
  .
  .
  C(19) = 0,1*19 + 4,4 (trilhões de cigarros vendidos em 1999)

  Observe que essa soma será então 0,1*(0+1+2+ ... +18+19) + 20*4,4 = 0,1*190 + 
88 = 107 trilhões de cigarros.

  É ISSO O QUE ESTÁ SENDO PEDIDO NO ENUNCIADO.

  Na minha opinião, esse gabarito está errado.

  Você conhece a origem dessa questão?

  Um abraço,

  João Luís.
  .
- Original Message - 
From: Gustavo Duarte 
To: obm-l@mat.puc-rio.br 
Sent: Tuesday, October 28, 2008 12:01 PM
Subject: [obm-l] Re: [obm-l] Re: [obm-l] Re: [obm-l] Re: [obm-l] PA com 
Função do 1 º grau.


Valeu  João !!! pensei assim: 1980 = C (0) =4,4 ;  1981 = C(1) =4,5 : 1982 
= C (2) = 4,6 ;  . ;  1999 = c (19) = 6,3 ; 2000 = C ( 20)  = 6,4. fiz até 
2000, pois ele fala no final de 1999. favor observar no final deste e-mail a 
fórmula que usei. Até mais
  - Original Message - 
  From: João Luís 
  To: obm-l@mat.puc-rio.br 
  Sent: Tuesday, October 28, 2008 1:16 PM
  Subject: [obm-l] Re: [obm-l] Re: [obm-l] Re: [obm-l] PA com Função do 1 º 
grau.


  Veja Gustavo, na verdade eu não resolvi o problema, apenas disse que essa 
PA deve ter 20 termos, pois são 20 anos, concorda? 1980 (inclusive) a 1999 
(inclusive).

  De onde você tirou que an = 6,4? Não entendi. DEsculpe-me se estou 
deixando passar algo óbvio, pois estou analisando as coisas meio na pressa aqui.

  Mais tarde, olho pro problema com toda a calma do mundo e posto uma 
mensagem aqui, ok?

  Um abraço a todos,

  João Luís.
- Original Message - 
From: Gustavo Duarte 
To: obm-l@mat.puc-rio.br 
Sent: Tuesday, October 28, 2008 10:11 AM
Subject: [obm-l] Re: [obm-l] Re: [obm-l] PA com Função do 1 º grau.


João, desculpe eu não ter entendido,para mim ainda não ficou claro... 
essa PA tem 20 termos é isso ?, qual seria o primeiro e qual seraia o último, 
usando na fórmula dá certo ?? onde estou errando ? pensei assim...se a1 = 4,4  
, a razão é 0,1 e an = 6,4 , n = ?? não seria 21 ? porém por outro lado vc tem 
razão ,do início de 1980 ate o final de 1999 realmente são 20 anos, O GABARITO 
TÀ CERTO È 108 MESMO ??  desde ja agradeço pela atenção !!
  - Original Message - 
  From: João Luís 
  To: obm-l@mat.puc-rio.br 
  Sent: Tuesday, October 28, 2008 7:30 AM
  Subject: [obm-l] Re: [obm-l] PA com Função do 1 º grau.


  De 1980 a 1999, são realmente 20 termos: 1999 - 1979 = 20.

  E porque  - 1979? Porque, como o 1980 está incluido nas condições 
do problema, não deve ser subtraído.

  Um abraço a todos,

  João Luís.


- Original Message - 
From: Gustavo Duarte 
To: Olimpíada 
Sent: Monday, October 27, 2008 10:59 PM
Subject: [obm-l] PA com Função do 1 º grau.


O número em trilhões de cigarros vendiddos, anualmente, é dado pela 
função C(t) = 0,1t + 4,4  , com t=0 correspondendo a 1980 ( contado a partir do 
início de 1980). Qual o número em trilhões de cigarros vendidos desde o início 
de 1980 até o final de 1999 ?

a) 109   b) 108   c) 107   d) 106  e) 105

O Gabarito dado é B) 108, fiz  e cheguei ao resultado assim , soma 
de PA,  [ ( 4,4 + 6,4)20 ] / 2 = 108, porém depois fazendo uma segunda análise 
discordei , pois essa PA não tem 20 termos e sim 21, porém o período dado é de 
20 anos, e aí ... alguém pode me ajudar na interpretação dessa questão ??? 
desde já agradeço.


  __ NOD32 3562 (20081028) Information __

  This message was checked by NOD32 antivirus system.
  http://www.eset.com



  __ NOD32 3562 (20081028) Information __

  This message was checked by NOD32 antivirus system.
  http://www.eset.com



  __ NOD32 3563 (20081028) Information __

  This message was checked by NOD32 antivirus system.
  http://www.eset.com


[obm-l] Somatório

2008-10-27 Por tôpico Gustavo Duarte
Tenho uma dúvida : O somatório de N, em que i varia de 1 até N é igual a ?? N 
ou N^N ou N^2, desde já agradeço qualquer ajuda.

[obm-l] PA com Função do 1 º grau.

2008-10-27 Por tôpico Gustavo Duarte
O número em trilhões de cigarros vendiddos, anualmente, é dado pela função C(t) 
= 0,1t + 4,4  , com t=0 correspondendo a 1980 ( contado a partir do início de 
1980). Qual o número em trilhões de cigarros vendidos desde o início de 1980 
até o final de 1999 ?

a) 109   b) 108   c) 107   d) 106  e) 105

O Gabarito dado é B) 108, fiz  e cheguei ao resultado assim , soma de PA,  [ ( 
4,4 + 6,4)20 ] / 2 = 108, porém depois fazendo uma segunda análise discordei , 
pois essa PA não tem 20 termos e sim 21, porém o período dado é de 20 anos, e 
aí ... alguém pode me ajudar na interpretação dessa questão ??? desde já 
agradeço.

Re: [obm-l] Fwd: help em logaritmo

2008-10-12 Por tôpico Gustavo Duarte
Olá que Marcelo, que tal passarmos todos esses log para base 2 ( mudança de 
base) seria ( quase) tudo simplificado e sobraia apenas LOG de 64 na base 2 que 
realmente é 6. 
Espero te ajudado!! abraços.
  - Original Message - 
  From: Marcelo Costa 
  To: obm-l@mat.puc-rio.br 
  Sent: Sunday, October 12, 2008 8:10 AM
  Subject: [obm-l] Fwd: help em logaritmo


  Alguém poderia me ajudar nesta questão que estou panguando, obrigado.



  (Mackenzie SP/2002/Janeiro)

  O produto (log2 3) × (log3 4) × (log4 5) ×…× (log63 64) é igual a:

  a)   log3 64

  b)   log2 63

  c)   2

  d)   4

  e)   6



  Gab: E






[obm-l] Fwd: F e G que comutam...

2008-10-07 Por tôpico Gustavo Simoes Araujo
 Ola Pessoal,

 Eu não estou conseguindo provar que se existem dois endomorphisms que
comutam f e g, dado um vetor proprio u de f, este sera também um vetor
proprio de g.  Sera que alguém poderia me dar uma mão ?

Abraços,

-- 
Gustavo Simões Araújo


[obm-l] F e G que comutam...

2008-10-06 Por tôpico Gustavo Simoes Araujo
Ola Pessoal,

 Eu não estou conseguindo provar que se existem dois endomorphisms que
comutam f e g, dado um vetor proprio u de f, este sera também um vetor
proprio de g.  Sera que alguém poderia me dar uma mão ?

Abraços,

-- 
Gustavo Simões Araújo


Re: [obm-l] BOTES

2008-09-07 Por tôpico Gustavo Duarte
Vão 2 e 1 ,  fica 2  e  volta 1, vão 8 e 4 ficam os dois e volta o 2( que tinha 
ficado da 1ª ida), e finalmente vão 1 e 2. tomados os tempos(maiores que estão 
em negrito) temos: 2 + 1 + 8 + 2 + 2  = 15h. Abraço e espero ter ajudadao!!  
Gustavo.
  - Original Message - 
  From: arkon 
  To: obm-l@mat.puc-rio.br 
  Sent: Friday, September 05, 2008 10:19 PM
  Subject: [obm-l] BOTES


   Pessoal, qual o macete para essa questão?

  Existem quatro botes numa margem de um rio; seus nomes são Oito, Quatro, Dois 
e Um, porque essas são as quantidades de horas que cada um deles demora para 
cruzar o rio. Pode-se atar um bote a outro, porém não mais de um, e então o 
tempo que demoram em cruzar é igual ao do mais lento dos botes. Um só 
marinheiro deve levar todos os botes até à outra margem do rio. Qual é o menor 
tempo necessário para completar o translado?



  Gabarito: 15 h.

  = 
Instru��es para entrar na lista, sair da lista e usar a lista em 
http://www.mat.puc-rio.br/~obmlistas/obm-l.html 
= 

[obm-l] Re: [obm-l] Re: [obm-l] Re: [obm-l] Re : [obm-l] Re: [obm-l] Re: [obm-l] semelh ança de triângulos

2008-08-21 Por tôpico Claudio Gustavo
  Acho que vc confundiu a construção, pois o segmento MX é exterior ao 
triângulo, logo X é exterior tanto a ABC quanto a MNP.
 
  Abraço.

--- Em qui, 21/8/08, luiz silva [EMAIL PROTECTED] escreveu:

De: luiz silva [EMAIL PROTECTED]
Assunto: [obm-l] Re: [obm-l] Re: [obm-l] Re: [obm-l] Re: [obm-l] Re: [obm-l] 
semelhança de triângulos
Para: obm-l@mat.puc-rio.br
Data: Quinta-feira, 21 de Agosto de 2008, 8:13







Cláudio,
 
Eu não entendi muito bem seu raciocínio, mas considere M pertencendo a BC,
 
No enunciado do problema, prolonga-se, a partir de O, o segmento MO em OX. Tal 
que MX = 1/2 altura traçada do vértice A.
 
O segmento NP é a base média do triângulo ABC, e dessa forma ele divide qqer 
ceviana do triângulo em segmentos iguais. Assim, o pontomédio da altura 
relativa a A esta no segmento NP. Como MX é paralelo a altura e com metade do 
seu comprimento, então X estará tb em NP e MX será perependicular a NP. Como 
isso vale para X,Y e Z, temos que estes pontos são os pés das alturas (que são 
os vértices dos triângulos órticos) do triângulo MNP, que é semelhante a ABC. 
Assim, XYZ é semelhante ao triângulo órtico de ABC.
 
 Abs
Felipe

--- Em qua, 20/8/08, Claudio Gustavo [EMAIL PROTECTED] escreveu:

De: Claudio Gustavo [EMAIL PROTECTED]
Assunto: [obm-l] Re: [obm-l] Re: [obm-l] Re: [obm-l] Re: [obm-l] semelhança de 
triângulos
Para: obm-l@mat.puc-rio.br
Data: Quarta-feira, 20 de Agosto de 2008, 20:45







  Olá.
  Bom:
1) X, Y e Z são os pontos médios das mediatrizes prolongadas? Vejamos, pois 
prolongamos OM em MX, então ficamos com o segmento OX que pertence à mediatriz 
do lado BC.
2) X, Y e Z são os pés das alturas de MNP? Na verdade os pontos X, Y e Z não 
pertencem aos segmentos MN, NP e PM. O triângulo XYZ não pode ser o órtico de 
MNP pq ele é maior. Na verdade ele é semelhante ao órtico de MNP, mas isso é o 
que o problema quer.
 
  Abraço.

--- Em qua, 20/8/08, luiz silva [EMAIL PROTECTED] escreveu:

De: luiz silva [EMAIL PROTECTED]
Assunto: [obm-l] Re: [obm-l] Re: [obm-l] Re: [obm-l] semelhança de triângulos
Para: obm-l@mat.puc-rio.br
Data: Quarta-feira, 20 de Agosto de 2008, 16:10







Com relação ao 1o. questionamento, é isso mesmo, mas temos que provar que a 
interseção dos lados do triângulo MNP com as mediatrizes são os pontos XYZ. E 
isso se dá, pq X, Y e Z tb são pontos médios das mediatrizes prolongadas com 
igual comprimento das alturas com relação as quais são paralelas (ficou 
confuso...mas desenhe e veja se fica mais claro).
 
Qto ao segundo questionamento, foi erro de escrita. Na realidade, o correto é : 
Como ABC é semelhante a MNP e XYZ são os pés das alturas de MNP (ou seja XYZ é 
o triângulo órtico de MNP), então XYZ é semelhante ao triângulo órtico de ABC.
 
Não sei se me fiz entender.
 
Abs
Felipe

--- Em qua, 20/8/08, Claudio Gustavo [EMAIL PROTECTED] escreveu:

De: Claudio Gustavo [EMAIL PROTECTED]
Assunto: [obm-l] Re: [obm-l] Re: [obm-l] semelhança de triângulos
Para: obm-l@mat.puc-rio.br
Data: Quarta-feira, 20 de Agosto de 2008, 11:38







  Oi Felipe.
  Obrigado pela solução, mas não entendi algumas partes que vou escrever. Se 
for possível, vc poderia esclarecer para mim.
 
 - Os lados do triângulo MNP cortam as alturas nos seus pontos médios e, 
consequentemente, também irão interceptar as mediatrizes em seus pontos médios.
  Mas as interseções das mediatrizes com o triângulo MNP não são os próprios 
vértices M, N e P?
 
 - Como MNP é semelhante a ABC, XYZ é semelhante a MNP.
  O triângulo MNP realmente é semelhante ao ABC, logo possuem todos os ângulos 
iguais (sejam eles A, B e C). Mas se XYZ é semelhante a MNP e MNP a ABC, pela 
transitividade XYZ seria semelhante a ABC. O problema pede para demosntrar que 
XYZ é semelhante ao órtico de ABC, que tem ângulos iguais a 180-2A, 180-2B e 
180-2C, respectivamente. Se XYZ é semelhante a MNP temos um caso particular de 
um triângulo ABC equilátero.
 
  Obrigado.
  Abraços,
Claudio Gustavo.

--- Em qua, 20/8/08, luiz silva [EMAIL PROTECTED] escreveu:

De: luiz silva [EMAIL PROTECTED]
Assunto: [obm-l] Re: [obm-l] semelhança de triângulos
Para: obm-l@mat.puc-rio.br
Data: Quarta-feira, 20 de Agosto de 2008, 9:21







Olá Cláudio,
 
Una os pontos médios MNP. Este triângulo é semelhante ao triângulo ABC, com 
seus lados sendo a metade dos lados deste triângulo.Agora prolongue ainda mais 
as mediatrizes traçadas, de modo que todas tenham o mesmo tamanho das 
alturas em relação as quais são paralelas.
 
Os lados do triângulo MNP cortam as alturas nos seus pontos médios e, 
consequentemente, também irão interceptar as mediatrizes em seus pontos médios.
 
Assim, os lados do triângulo MNP irão conter os pontos X,Y e Z, sendo estes 
pontos os pés das alturas do triângulo MNP. Ou seja, o triângulo XYZ é o 
triângulo órtico do triângulo MNP. Como MNP é semelhante a ABC, XYZ é 
semelhante a MNP.
 
Abs
Felipe
--- Em ter, 19/8/08, Claudio Gustavo [EMAIL PROTECTED] escreveu:

De: Claudio Gustavo [EMAIL PROTECTED]
Assunto

[obm-l] Re: [obm-l] Re: [obm-l] semelhança de triâ ngulos

2008-08-20 Por tôpico Claudio Gustavo
  Oi Felipe.
  Obrigado pela solução, mas não entendi algumas partes que vou escrever. Se 
for possível, vc poderia esclarecer para mim.
 
 - Os lados do triângulo MNP cortam as alturas nos seus pontos médios e, 
consequentemente, também irão interceptar as mediatrizes em seus pontos médios.
  Mas as interseções das mediatrizes com o triângulo MNP não são os próprios 
vértices M, N e P?
 
 - Como MNP é semelhante a ABC, XYZ é semelhante a MNP.
  O triângulo MNP realmente é semelhante ao ABC, logo possuem todos os ângulos 
iguais (sejam eles A, B e C). Mas se XYZ é semelhante a MNP e MNP a ABC, pela 
transitividade XYZ seria semelhante a ABC. O problema pede para demosntrar que 
XYZ é semelhante ao órtico de ABC, que tem ângulos iguais a 180-2A, 180-2B e 
180-2C, respectivamente. Se XYZ é semelhante a MNP temos um caso particular de 
um triângulo ABC equilátero.
 
  Obrigado.
  Abraços,
Claudio Gustavo.

--- Em qua, 20/8/08, luiz silva [EMAIL PROTECTED] escreveu:

De: luiz silva [EMAIL PROTECTED]
Assunto: [obm-l] Re: [obm-l] semelhança de triângulos
Para: obm-l@mat.puc-rio.br
Data: Quarta-feira, 20 de Agosto de 2008, 9:21







Olá Cláudio,
 
Una os pontos médios MNP. Este triângulo é semelhante ao triângulo ABC, com 
seus lados sendo a metade dos lados deste triângulo.Agora prolongue ainda mais 
as mediatrizes traçadas, de modo que todas tenham o mesmo tamanho das 
alturas em relação as quais são paralelas.
 
Os lados do triângulo MNP cortam as alturas nos seus pontos médios e, 
consequentemente, também irão interceptar as mediatrizes em seus pontos médios.
 
Assim, os lados do triângulo MNP irão conter os pontos X,Y e Z, sendo estes 
pontos os pés das alturas do triângulo MNP. Ou seja, o triângulo XYZ é o 
triângulo órtico do triângulo MNP. Como MNP é semelhante a ABC, XYZ é 
semelhante a MNP.
 
Abs
Felipe
--- Em ter, 19/8/08, Claudio Gustavo [EMAIL PROTECTED] escreveu:

De: Claudio Gustavo [EMAIL PROTECTED]
Assunto: [obm-l] semelhança de triângulos
Para: obm-l obm-l@mat.puc-rio.br
Data: Terça-feira, 19 de Agosto de 2008, 20:40







  Oi.
  Gostaria de ajuda no problema abaixo. Se for possível, dando a solução usando 
apenas argumenos de geometria plana (sem auxílio de complexos ou analítica).
 
 - Sejam M, N e P os pontos médios dos lados de um triângulo ABC acutângulo de 
circuncentro O. Prolongue MO, NO e PO, a partir de O, até X, Y e Z, 
respectivamente, tais que MX, NY e PZ tenham comprimentos respectivamente 
iguais às metades das alturas do triângulo a partir dos vértices A, B e C. 
Prove que o triângulo XYZ é semelhante ao triângulo órtico de ABC.
 
  Obrigado.


Novos endereços, o Yahoo! que você conhece. Crie um email novo com a sua cara 
@ymail.com ou @rocketmail.com.


Novos endereços, o Yahoo! que você conhece. Crie um email novo com a sua cara 
@ymail.com ou @rocketmail.com.


  Novos endereços, o Yahoo! que você conhece. Crie um email novo com a sua 
cara @ymail.com ou @rocketmail.com.
http://br.new.mail.yahoo.com/addresses

[obm-l] Re: [obm-l] Re: [obm-l] Re: [obm-l] Re : [obm-l] semelhança de triângulos

2008-08-20 Por tôpico Claudio Gustavo
  Olá.
  Bom:
1) X, Y e Z são os pontos médios das mediatrizes prolongadas? Vejamos, pois 
prolongamos OM em MX, então ficamos com o segmento OX que pertence à mediatriz 
do lado BC.
2) X, Y e Z são os pés das alturas de MNP? Na verdade os pontos X, Y e Z não 
pertencem aos segmentos MN, NP e PM. O triângulo XYZ não pode ser o órtico de 
MNP pq ele é maior. Na verdade ele é semelhante ao órtico de MNP, mas isso é o 
que o problema quer.
 
  Abraço.

--- Em qua, 20/8/08, luiz silva [EMAIL PROTECTED] escreveu:

De: luiz silva [EMAIL PROTECTED]
Assunto: [obm-l] Re: [obm-l] Re: [obm-l] Re: [obm-l] semelhança de triângulos
Para: obm-l@mat.puc-rio.br
Data: Quarta-feira, 20 de Agosto de 2008, 16:10







Com relação ao 1o. questionamento, é isso mesmo, mas temos que provar que a 
interseção dos lados do triângulo MNP com as mediatrizes são os pontos XYZ. E 
isso se dá, pq X, Y e Z tb são pontos médios das mediatrizes prolongadas com 
igual comprimento das alturas com relação as quais são paralelas (ficou 
confuso...mas desenhe e veja se fica mais claro).
 
Qto ao segundo questionamento, foi erro de escrita. Na realidade, o correto é : 
Como ABC é semelhante a MNP e XYZ são os pés das alturas de MNP (ou seja XYZ é 
o triângulo órtico de MNP), então XYZ é semelhante ao triângulo órtico de ABC.
 
Não sei se me fiz entender.
 
Abs
Felipe

--- Em qua, 20/8/08, Claudio Gustavo [EMAIL PROTECTED] escreveu:

De: Claudio Gustavo [EMAIL PROTECTED]
Assunto: [obm-l] Re: [obm-l] Re: [obm-l] semelhança de triângulos
Para: obm-l@mat.puc-rio.br
Data: Quarta-feira, 20 de Agosto de 2008, 11:38







  Oi Felipe.
  Obrigado pela solução, mas não entendi algumas partes que vou escrever. Se 
for possível, vc poderia esclarecer para mim.
 
 - Os lados do triângulo MNP cortam as alturas nos seus pontos médios e, 
consequentemente, também irão interceptar as mediatrizes em seus pontos médios.
  Mas as interseções das mediatrizes com o triângulo MNP não são os próprios 
vértices M, N e P?
 
 - Como MNP é semelhante a ABC, XYZ é semelhante a MNP.
  O triângulo MNP realmente é semelhante ao ABC, logo possuem todos os ângulos 
iguais (sejam eles A, B e C). Mas se XYZ é semelhante a MNP e MNP a ABC, pela 
transitividade XYZ seria semelhante a ABC. O problema pede para demosntrar que 
XYZ é semelhante ao órtico de ABC, que tem ângulos iguais a 180-2A, 180-2B e 
180-2C, respectivamente. Se XYZ é semelhante a MNP temos um caso particular de 
um triângulo ABC equilátero.
 
  Obrigado.
  Abraços,
Claudio Gustavo.

--- Em qua, 20/8/08, luiz silva [EMAIL PROTECTED] escreveu:

De: luiz silva [EMAIL PROTECTED]
Assunto: [obm-l] Re: [obm-l] semelhança de triângulos
Para: obm-l@mat.puc-rio.br
Data: Quarta-feira, 20 de Agosto de 2008, 9:21







Olá Cláudio,
 
Una os pontos médios MNP. Este triângulo é semelhante ao triângulo ABC, com 
seus lados sendo a metade dos lados deste triângulo.Agora prolongue ainda mais 
as mediatrizes traçadas, de modo que todas tenham o mesmo tamanho das 
alturas em relação as quais são paralelas.
 
Os lados do triângulo MNP cortam as alturas nos seus pontos médios e, 
consequentemente, também irão interceptar as mediatrizes em seus pontos médios.
 
Assim, os lados do triângulo MNP irão conter os pontos X,Y e Z, sendo estes 
pontos os pés das alturas do triângulo MNP. Ou seja, o triângulo XYZ é o 
triângulo órtico do triângulo MNP. Como MNP é semelhante a ABC, XYZ é 
semelhante a MNP.
 
Abs
Felipe
--- Em ter, 19/8/08, Claudio Gustavo [EMAIL PROTECTED] escreveu:

De: Claudio Gustavo [EMAIL PROTECTED]
Assunto: [obm-l] semelhança de triângulos
Para: obm-l obm-l@mat.puc-rio.br
Data: Terça-feira, 19 de Agosto de 2008, 20:40







  Oi.
  Gostaria de ajuda no problema abaixo. Se for possível, dando a solução usando 
apenas argumenos de geometria plana (sem auxílio de complexos ou analítica).
 
 - Sejam M, N e P os pontos médios dos lados de um triângulo ABC acutângulo de 
circuncentro O. Prolongue MO, NO e PO, a partir de O, até X, Y e Z, 
respectivamente, tais que MX, NY e PZ tenham comprimentos respectivamente 
iguais às metades das alturas do triângulo a partir dos vértices A, B e C. 
Prove que o triângulo XYZ é semelhante ao triângulo órtico de ABC.
 
  Obrigado.


Novos endereços, o Yahoo! que você conhece. Crie um email novo com a sua cara 
@ymail.com ou @rocketmail.com.


Novos endereços, o Yahoo! que você conhece. Crie um email novo com a sua cara 
@ymail.com ou @rocketmail.com.


Novos endereços, o Yahoo! que você conhece. Crie um email novo com a sua cara 
@ymail.com ou @rocketmail.com.


Novos endereços, o Yahoo! que você conhece. Crie um email novo com a sua cara 
@ymail.com ou @rocketmail.com.


  Novos endereços, o Yahoo! que você conhece. Crie um email novo com a sua 
cara @ymail.com ou @rocketmail.com.
http://br.new.mail.yahoo.com/addresses

[obm-l] semelhança de triângulos

2008-08-19 Por tôpico Claudio Gustavo
  Oi.
  Gostaria de ajuda no problema abaixo. Se for possível, dando a solução usando 
apenas argumenos de geometria plana (sem auxílio de complexos ou analítica).
 
 - Sejam M, N e P os pontos médios dos lados de um triângulo ABC acutângulo de 
circuncentro O. Prolongue MO, NO e PO, a partir de O, até X, Y e Z, 
respectivamente, tais que MX, NY e PZ tenham comprimentos respectivamente 
iguais às metades das alturas do triângulo a partir dos vértices A, B e C. 
Prove que o triângulo XYZ é semelhante ao triângulo órtico de ABC.
 
  Obrigado.


  Novos endereços, o Yahoo! que você conhece. Crie um email novo com a sua 
cara @ymail.com ou @rocketmail.com.
http://br.new.mail.yahoo.com/addresses

Re: [obm-l] Resolvendo um sisteminha

2008-07-04 Por tôpico Gustavo Duarte
Observe caro Marcelo, para a = 1, teremos duas expressões   iguais com 
resultados diferentes,ou seja , para a = 1, nenhum par ordenado (x,y) torna 
verdadeira as duas equações do sistema !! espero ter ajuadado .
  - Original Message - 
  From: Marcelo Gomes 
  To: obm-l@mat.puc-rio.br 
  Sent: Friday, July 04, 2008 10:31 AM
  Subject: [obm-l] Resolvendo um sisteminha


  Olá pessoal bom dia.

  Um sisteminha para quem puder ajudar :

  Considere o sistema :

  x + ay = 1
  x +  y = 4 

  Para quais valores de a o sistema não tem solução ?

  Um abraço, Gomes.


[obm-l] Congruências

2008-06-17 Por tôpico Gustavo Simoes Araujo
Ola Pessoal,

  Será que alguém poderia me ajudar com esta questão ?

Prove que 19^19 + 69^69 é divisível por 44.

Abraços,

-- 
Gustavo Simões Araújo


Re: [obm-l] Congruências

2008-06-17 Por tôpico Gustavo Simoes Araujo
Valeu gente. Obrigado.

-- 
Gustavo Simões Araújo


Re: [obm-l] Sequência convergente ?

2008-06-14 Por tôpico Gustavo Simoes Araujo
Valeu Bruno !

 Abraços,

-- 
Gustavo Simões Araújo


Re: [obm-l] outra de sequencia

2008-06-13 Por tôpico Gustavo Simoes Araujo
Ola Luciana,

Então acho que você pode resolver esta questão desta maneira:

a_n= sqrt(n+h) - sqrt(n) = ( sqrt(n+h) - sqrt(n) ) *  (sqrt(n+h) + sqrt(n)
)/ (sqrt(n+h) + sqrt(n) )

a_n= ( (n+h) - n )/  (sqrt(n+h) + sqrt(n) ) = h /  (sqrt(n+h) + sqrt(n) )
tendendo este limite para zero quando n tende para o infinito.

Abraços,


-- 
Gustavo Simões Araújo


Re: [obm-l] Re:

2008-06-13 Por tôpico Gustavo Duarte
Desculpe Rafael, não estou conseguindo entender legal esse começo:
  escrever 989 como 1e11 - 1e5 - 1, ao quadrado isso vale:

  1e22 - 2e16 - 2e11 + 1e10 + 2e5 + 1. e daí para frente eu nem  continuei 
...tens como me ajudar ?

  ge - 
  From: Rafael Ando 
  To: obm-l@mat.puc-rio.br 
  Sent: Thursday, June 12, 2008 6:44 AM
  Subject: [obm-l] Re:


  1) Escreva 989 como 1e11 - 1e5 - 1, ao quadrado isso vale:

  1e22 - 2e16 - 2e11 + 1e10 + 2e5 + 1.

  Vamos tentar entender essa expressao, fazendo operacao por operacao 
(desculpem pela falta de acentuacao, estou em um teclado que nao tem acentos e 
cedilha...): 1e22 - 2e16 sao 22-16-1=5 noves, seguido de um 8, e 16 zeros 
(98).

  A proxima operacao eh fazer esse numero -2e11. O numero 8 transforma-se em 7, 
o zero na posicao 11 vira 8 e os 16-11-1 = 4 anteriores viram 9, os demais 
continuam 0. Entao temos o numero 978000. As demais operacoes 
sao somas nos 0s, nao vai aparecer nenhum 9 e nem destruir os que ja temos 
entao a resposta eh 9.

  Letra B.

  2) Podemos ver que 15x^2 eh congruente a (ie, termina com) 0 ou 5 mod10. y^2, 
com y inteiro, eh congruente a 0,1,4,5,6 ou 9, entao 7y^2 eh congruente a 
0,7,8,5,2 ou 3. Pra resposta ser 9, teriamos que ter 7y^2 congruente a 1 (para 
o caso 15x^2 conguente a 0) ou 6 (para o congruente a 5), nenhum deles eh 
possivel. Logo, essa equacao nao tem solucao (diofantina).

  Letra A.


  On Mon, Apr 7, 2008 at 5:47 AM, fgb1 [EMAIL PROTECTED] wrote:

Amigos... Tõ precisando de ajuda em 2 problemas:

1) Quantos noves existem na representação decimal de 989^2?

a) 7
b) 9
c) 11
d) 13
e) 15

2) Quantos inteiros x e y existem tais que 15x^2 - 7y^2 = 9/

a) 0
b) 1
c) 2
d) 3
e) 4

Desde já agradeço


=
Instruções para entrar na lista, sair da lista e usar a lista em
http://www.mat.puc-rio.br/~obmlistas/obm-l.html
=




  -- 
  Rafael 

Re: [obm-l] Re:

2008-06-13 Por tôpico Gustavo Duarte

Valeu !! tudo entendido obg.
- Original Message - 
From: Maurício Collares [EMAIL PROTECTED]

To: obm-l@mat.puc-rio.br
Sent: Friday, June 13, 2008 9:04 PM
Subject: Re: [obm-l] Re:



A notação 2e11 significa 2 * 10^11, onde o ^ significa elevado a. Ou
seja, traduzindo, o que o Rafael disse foi:

Escrevendo 989 como 10^11 - 10^5 - 1, temos que:

(989)^2 = (10^11 - 10^5 - 1)^2
= 10^22 - 10^16 - 10^11 - 10^16 + 10^10 + 10^5 - 10^11 + 10^5 + 1
= 10^22 - 2*10^16 - 2*10^11 + 10^10 + 2*10^5 + 1

--
Abraços,
Maurício

2008/6/13 Gustavo Duarte [EMAIL PROTECTED]:


Desculpe Rafael, não estou conseguindo entender legal esse começo:

escrever 989 como 1e11 - 1e5 - 1, ao quadrado isso vale:

1e22 - 2e16 - 2e11 + 1e10 + 2e5 + 1. e daí para frente eu nem  continuei 
...tens como me ajudar ?


=
Instruções para entrar na lista, sair da lista e usar a lista em
http://www.mat.puc-rio.br/~obmlistas/obm-l.html
=



=
Instruções para entrar na lista, sair da lista e usar a lista em
http://www.mat.puc-rio.br/~obmlistas/obm-l.html
=


[obm-l] Sequência convergente ?

2008-06-12 Por tôpico Gustavo Simoes Araujo
Ola Pessoal,

   Estou tentando fazer um problema e não consigo. Será que vocês
poderiam me ajudar ? O problema é o seguinte...

*a) - Seja f(x) = x^2 -1. Mostre que f admite um ponto fixo no domínio D a
definir. Seja a sequência u_n+1=f(u_n), u_0 pertencente à D. Qual a ordem de
convergência de u_n ?*

   Meu problema é básico, não consigo provar que a sequência é
convergente. Se eu uso como valor inicial u_0 um numero entre  -(1+
raiz(5))/2  u_0  (1+raiz(5))/2  meu problema converge para os valores 0
e -1, que se alternam a cada interação. Se eu escolher um numero maior que
mod (u_0)  (1+raiz(5))/2 a sequência diverge Estou fazendo algum erro ?
Ou não tem sentido pedir para calcular a ordem de convergência para esta
sequência ?

Valeu abraços,

-- 
Gustavo Simões Araújo


[obm-l] analise comb.(difícil)

2008-06-05 Por tôpico Gustavo Duarte
Qualquer ajuda é bem vinda, tive dificuldade nas duas !!desde já agradeço.

1)Escolhemos 5 números ,sem repetição, dentre os inteiros de 1 a 20.Calcule 
quantas escolhas distintas podemser feitas, sabendo que ao menos dois dos 5 
números selecioneodos devem deixar o mesmo resto quando dividido por 5.

2) Escrevem-se números de 5 algarismo (inclusive começando por Zero)) em 
cartões. Como 0, 1 e 8 não se alteram de cabeça para baixo e como o 6 de cabeça 
para baixo vira 9, um só cartão pode representar dois números ( por exemplo 
06198 e 86190). Qual o número mínimo de cartões para representar todos os 
números de 5 algarismo ?


[obm-l] Re: [obm-l] Re: [obm-l] analise comb.(difíci l)

2008-06-05 Por tôpico Gustavo Duarte
Puxa Carlos , muitíssimo obrigado, eu não tinha notado nada disso, por esse 
caminho que fizestes ficou bem legal !!! VALEU !
  - Original Message - 
  From: Carlos Gomes 
  To: obm-l@mat.puc-rio.br 
  Sent: Thursday, June 05, 2008 10:08 PM
  Subject: [obm-l] Re: [obm-l] analise comb.(difícil)


  Gustavo,  vamos arrumar os números de 1 a 20 da seguinte forma:

  12345
  6789   10
  111213  14  15
  161718  19  20


  Note que numa mesma coluna estão os números que deixam o mesmo resto quando 
divididos por 5.Como queremos escoher, dentre  os 20 números, 5 números 
distintos onde pelo menos 2 deixam o mesmo resto quando divididos por 5 , temos 
:

  N° total de modos de escolher 5 números distintos de 1 a 20   === 
C(20,5)=15504

  N° total de modos de escolher 5 números com restos diferentes (quando 
divididos por 5) == Para isto você deve pegar um em cada coluna o que pode ser 
feito de 4.4.4.4.4=1024 modos

  assim o nímero de possibilidade procuradas é 15504-1024=14480.



  Gustavo,

  Note que esxistem três tipos de números

  I. Os que virados de cabeça para baixo não representam nenhum número (por 
exemplo, 45.189)

  II. Os que virados de cabeça para baixo representam o mesmo número (por 
exemplo, 86198)

  III. Os que virados de cabeça para baixo representam números diferentes ( por 
exemplo, 66810)

  Note que  os cartões capazes de economizar são os terceiro tipo pois esses 
podem representar dois números. Vamos contar quantos são os números que podem 
ser representados por um mesmo cartão. Para isso perceba que os possíveis 
algarismos de serem virados e ainda assim representarem o mesmo número no 
cartão são 0,1,6,8 e 9. Assim tems 5.5.5.5.5=3125 possibilidades de fabricar um 
número de 5 algarismos com os digitos 0,1,6,8 e 9. Mas destes 3125 números 
quantos são do tipo II?

  Note que para ser do tipo II , as casas das extremidades devem der 0 e 0  ou  
1 e 1   ou  6 e 9   ou 8  e  8  ou 9 e 6, tendo assim 5 possibilidades, para a 
segunda e quarta casas as mesmas 5 possibilidade e finalmente para a casa 
central 0 , 1 ou 8  que são 3 possibilidades, havendo portanto 5.5.3=75 cartões 
que de cabeça para baixo podem representar o mesmo número. Assim os números do 
tipo III são 3125-75=3050. Como cada cartão pode representar dois deses números 
segue que nesta modalidade o número de cartões necessãrios é 3050/2=1525.

  Assim o número mínimo de cartões para representat os 10 números de cinco 
algarismos é 100.000-1525=98475


  Valeu,

  Cgomes

  - Original Message - 
From: Gustavo Duarte 
To: Olimpíada 
Sent: Thursday, June 05, 2008 8:28 PM
Subject: [obm-l] analise comb.(difícil)


Qualquer ajuda é bem vinda, tive dificuldade nas duas !!desde já agradeço.

1)Escolhemos 5 números ,sem repetição, dentre os inteiros de 1 a 20.Calcule 
quantas escolhas distintas podemser feitas, sabendo que ao menos dois dos 5 
números selecioneodos devem deixar o mesmo resto quando dividido por 5.

2) Escrevem-se números de 5 algarismo (inclusive começando por Zero)) em 
cartões. Como 0, 1 e 8 não se alteram de cabeça para baixo e como o 6 de cabeça 
para baixo vira 9, um só cartão pode representar dois números ( por exemplo 
06198 e 86190). Qual o número mínimo de cartões para representar todos os 
números de 5 algarismo ?



__ Informação do NOD32 IMON 1.1189 (20050808) __

Esta mensagem foi verificada pelo NOD32 sistema antivírus
http://www.eset.com.br



  __ NOD32 3162 (20080605) Information __

  This message was checked by NOD32 antivirus system.
  http://www.eset.com


[obm-l] 101001

2008-05-15 Por tôpico Gustavo Duarte
Recebi esta questão, não conhecia , gostei e achei legal deividir com vcs :
Considere a sequência númerica : 10100100011...,é correto afirmar que há um 
número 1 na posição:
a) 168
b) 169
c) 170
d) 171
e) 172

GAB. D

[obm-l] Ajuda a Provar

2008-04-29 Por tôpico Gustavo Souza
alguem por favor me ajudaria a provar essa igualdade...
   
   
  1³ + 2³ + 3³ + ... + n³ = (1+2+3+...+n)²
   
  Abraços

   
-
Abra sua conta no Yahoo! Mail, o único sem limite de espaço para armazenamento! 

Re: [obm-l] Prova do ITA

2008-04-22 Por tôpico Gustavo Souza
Provas de matematica de 89 ate 2005
   
  http://www.rumoaoita.com/materiais/elite/provas_mat_%20ita_89-05.pdf
  

João Gabriel Preturlan [EMAIL PROTECTED] escreveu:
Gostaria de saber se alguém tem a prova de matemática do ITA de 
1989 e poderia me mandar ou indicar um link.
   
  Grato
   
  JG


  No virus found in this outgoing message.
Checked by AVG.
Version: 7.5.524 / Virus Database: 269.23.3/1390 - Release Date: 21/04/2008 
16:23



   
-
Abra sua conta no Yahoo! Mail, o único sem limite de espaço para armazenamento! 

[obm-l] Re: [obm-l] Exercício - Fabricação de bicicletas (alterado)

2008-04-20 Por tôpico Gustavo Duarte
Vamos lá dória:

96 / ( b +2 )   + 84 / b = 26, com b = ao preço unitário do produto b, ou seja 
ao didivir o preço total pelo preço unitário temos a quantidade comprada , que 
no problema soma um total de 26, a partir daí é tita o mínio e encontra b = 6 
,logo o preço de a= 8 e R$ 96 dividido por R$ 8 é igual a 12. Espero ter 
ajudado, abraços.
  - Original Message - 
  From: Dória 
  To: obm-l@mat.puc-rio.br 
  Sent: Wednesday, April 16, 2008 12:31 PM
  Subject: [obm-l] Exercício - Fabricação de bicicletas (alterado)


  Olá!

  Por favor, me ajudem nesse exercício:

  Para a fabricação de bicicletas, uma empresa comprou unidades do produto A, 
pagando R$96,00, e unidades do produto B, pagando R$84,00. Sabendo-se que o 
total de unidades compradas foi de 26 e que o preço unitário do produto A 
excede em R$2,00 o preço unitário do produto B, determine o número de unidades 
de A que foi comprado.

  Resposta: 12

  Muito obrigado, []'s

Re: [obm-l] Re: [obm-l] RES: [obm-l] Aplicaç ão de Matemática à Física

2008-04-15 Por tôpico Gustavo Souza
uhum...pesquisei agora eu achei, a pergunta é do ITA mesmo mas não sei de que 
ano...
   
  Esquece o que eu tinha falado anteriormente...
   
  Abraços
   
  

arkon [EMAIL PROTECTED] escreveu:
Se eu não me engano essa questão é do ITA-1966.
   
  ARKON
   
   Obrigado pela ajuda João
   Então, o exercício foi retirado de uma apostila de reforço para o ITA, de 
um cursinho preparatório ( ETAPA - disse o nome somente porque o exercício foi 
criado por eles e não retirado de alguma prova )

   Novamente Obrigado...

   Gustavo
   

João Gabriel Preturlan [EMAIL PROTECTED] escreveu:
Hey...
   
  Então vou chamar o mais rápido de A e o mais lento de B... vou chamar a 
distância entre as margens de d e como v=dist/tempo; então 
tempo=distancia/velocidade.
   
  Agora, digamos que eles se encontrem pela primeira vez num instante t.
  Então t = (distancia percorrida por A)/Va que é t = (d – 720)/Va
  Além disso podemos dizer que t = (distancia percorrida por B)/Vb que é t = 
720/Vb
  Igualando os dois t, temos que Va/Vb=(d – 720)/720
   
  Agora digamos que num instante t’ eles se encontrem novamente. Como o 
intervalo de tempo em que ambos ficam em repouso é igual, então este repouso 
não altera a solução. Mesmo que o tempo em que eles tivessem parados fosse 
desconhecidos, a partir do momento que ele é igual tanto para A quanto para B, 
nada muda.
  Logo como distância percorrida por A até este momento é a travessia completa 
da margem mais d-400; então o A percorre d + d – 400 = 2d – 400;
  Logo t’ = (2d – 400)/Va
  Logo, a distância percorrida por B até este instante é um travessia completa 
mais 400 metros; então B percorre d + 400;
  Então t’ = (d + 400)/Vb
  Igualando os dois t’, temos: Va/Vb=(2d – 400)/(d+400)
   
  Igualando Va/Vb nos dois casos tomos a igualdade: (d – 720)/720=(2d – 
400)/(d+400)
  Multiplicando ‘em cruz’ temos: d^2 – 1760d=0 Assim d = 1760
   
  Então se você montar um gráfico espaço x tempo fica mais fácil de 
visualizar estas relações...
  Espero que eu sido claro na solução.
  Mas muito legal o problema, onde você arrumou ele?
   
  Abç.
   
  JG.
De: [EMAIL PROTECTED] [mailto:[EMAIL PROTECTED] Em nome de Gustavo Souza
Enviada em: quinta-feira, 10 de abril de 2008 00:43
Para: obm-l@mat.puc-rio.br
Assunto: [obm-l] Aplicação de Matemática à Física

   
  

 Quem puder resolver esse exercicio por favor, pois estou tendo enormes 
dificuldades...

  

 Dois barcos partem, num mesmo instante, de lados opostos de um rio de 
margens paralelas. Viajam,cada qual, perpendicularmente às margens, com 
velocidades constantes. Supondo que um deles é mais rápido que o outro, eles se 
cruzam num ponto situado a 720 metros da margem mais próxima. Completada a 
travessia, cada barco fica parado no respectivo cais por 10 minutos. Na volta 
eles se cruzam a 400 metros da outra margem. Qual a largura do rio?

  

  

  

  

  

  

 Resposta  1760 metros

 

-
  
  Abra sua conta no Yahoo! Mail, o único sem limite de espaço para 
armazenamento! 
   No virus found in this incoming message.
Checked by AVG.
Version: 7.5.519 / Virus Database: 269.22.10/1367 - Release Date: 09/04/2008 
07:10


   No virus found in this outgoing message.
Checked by AVG.
Version: 7.5.519 / Virus Database: 269.22.10/1367 - Release Date: 09/04/2008 
07:10



-
  Abra sua conta no Yahoo! Mail, o único sem limite de espaço para 
armazenamento! 

 between -00-00 and -99-99   
-
Abra sua conta no Yahoo! Mail, o único sem limite de espaço para armazenamento! 

[obm-l] Re: [obm-l] Análise Combinatória: dúvida...

2008-04-12 Por tôpico Gustavo Duarte
Acho que está certo, eu tb resolveria assim !!
  - Original Message - 
  From: cleber vieira 
  To: obm-l@mat.puc-rio.br 
  Sent: Wednesday, April 09, 2008 9:53 PM
  Subject: [obm-l] Análise Combinatória: dúvida...


  Amigos gostaria da opinião de vcs sobre a resolução que fiz do seguinte 
problema:

  Um dia pode ter uma de sete classificações: MB(muito bom), B(bom), O(ótimo), 
P(péssimo), S(sofrível) e T(terrivel). Os dias de uma semana são: domingo, 
segunda, terça, quarta,quinta, sexta e sábado. Duas semanas se dizem distintas 
se dois dias de mesmo nome  têm classificações distintas. Quantas semanas 
distintas, segundo o critério dado, existem?

  a) 7!b) 7^2c) 7*7!d) 7^7e) (7^7)!

  Minha resolução foi a seguinte:
  segunda = 7 possib.
  terça =  7 possib.
  quarta =7 possib.
  quinta =7 possib.
  sexta = 7 possib. 
  sábado =  7 possib.
  domingo =7 possib.
  Como cada classificação de um dia da semana é independente dos outros dias e 
como cada dia da semana tem 7 possibilidades, teremos 7^7 semanas distintas.
  Desde já agradeço.




--
  Abra sua conta no Yahoo! Mail, o único sem limite de espaço para 
armazenamento! 

Re: [obm-l] RES: [obm-l] Aplicação de Mate mática à Física

2008-04-10 Por tôpico Gustavo Souza
Obrigado pela ajuda João
  Então, o exercício foi retirado de uma apostila de reforço para o ITA, de um 
cursinho preparatório ( ETAPA - disse o nome somente porque o exercício foi 
criado por eles e não retirado de alguma prova )
   
  Novamente Obrigado...
   
  Gustavo
  

João Gabriel Preturlan [EMAIL PROTECTED] escreveu:
v\:* {behavior:url(#default#VML);}  o\:* {behavior:url(#default#VML);}  
w\:* {behavior:url(#default#VML);}  .shape {behavior:url(#default#VML);}
Hey...
   
  Então vou chamar o mais rápido de A e o mais lento de B... vou chamar a 
distância entre as margens de d e como v=dist/tempo; então 
tempo=distancia/velocidade.
   
  Agora, digamos que eles se encontrem pela primeira vez num instante t.
  Então t = (distancia percorrida por A)/Va que é t = (d – 720)/Va
  Além disso podemos dizer que t = (distancia percorrida por B)/Vb que é t = 
720/Vb
  Igualando os dois t, temos que Va/Vb=(d – 720)/720
   
  Agora digamos que num instante t’ eles se encontrem novamente. Como o 
intervalo de tempo em que ambos ficam em repouso é igual, então este repouso 
não altera a solução. Mesmo que o tempo em que eles tivessem parados fosse 
desconhecidos, a partir do momento que ele é igual tanto para A quanto para B, 
nada muda.
  Logo como distância percorrida por A até este momento é a travessia completa 
da margem mais d-400; então o A percorre d + d – 400 = 2d – 400;
  Logo t’ = (2d – 400)/Va
  Logo, a distância percorrida por B até este instante é um travessia completa 
mais 400 metros; então B percorre d + 400;
  Então t’ = (d + 400)/Vb
  Igualando os dois t’, temos: Va/Vb=(2d – 400)/(d+400)
   
  Igualando Va/Vb nos dois casos tomos a igualdade: (d – 720)/720=(2d – 
400)/(d+400)
  Multiplicando ‘em cruz’ temos: d^2 – 1760d=0 Assim d = 1760
   
  Então se você montar um gráfico espaço x tempo fica mais fácil de 
visualizar estas relações...
  Espero que eu sido claro na solução.
  Mas muito legal o problema, onde você arrumou ele?
   
  Abç.
   
  JG.
De: [EMAIL PROTECTED] [mailto:[EMAIL PROTECTED] Em nome de Gustavo Souza
Enviada em: quinta-feira, 10 de abril de 2008 00:43
Para: obm-l@mat.puc-rio.br
Assunto: [obm-l] Aplicação de Matemática à Física

   
 

Quem puder resolver esse exercicio por favor, pois estou tendo enormes 
dificuldades...

 

Dois barcos partem, num mesmo instante, de lados opostos de um rio de 
margens paralelas. Viajam,cada qual, perpendicularmente às margens, com 
velocidades constantes. Supondo que um deles é mais rápido que o outro, eles se 
cruzam num ponto situado a 720 metros da margem mais próxima. Completada a 
travessia, cada barco fica parado no respectivo cais por 10 minutos. Na volta 
eles se cruzam a 400 metros da outra margem. Qual a largura do rio?

 

 

 

 

 

 

Resposta  1760 metros



-
  
  Abra sua conta no Yahoo! Mail, o único sem limite de espaço para 
armazenamento! 
  No virus found in this incoming message.
Checked by AVG.
Version: 7.5.519 / Virus Database: 269.22.10/1367 - Release Date: 09/04/2008 
07:10


  No virus found in this outgoing message.
Checked by AVG.
Version: 7.5.519 / Virus Database: 269.22.10/1367 - Release Date: 09/04/2008 
07:10



   
-
Abra sua conta no Yahoo! Mail, o único sem limite de espaço para armazenamento! 

[obm-l] Aplicação de Matemática à Física

2008-04-09 Por tôpico Gustavo Souza
 
  Quem puder resolver esse exercicio por favor, pois estou tendo enormes 
dificuldades...
   
  Dois barcos partem, num mesmo instante, de lados opostos de um rio de margens 
paralelas. Viajam,cada qual, perpendicularmente às margens, com velocidades 
constantes. Supondo que um deles é mais rápido que o outro, eles se cruzam num 
ponto situado a 720 metros da margem mais próxima. Completada a travessia, cada 
barco fica parado no respectivo cais por 10 minutos. Na volta eles se cruzam a 
400 metros da outra margem. Qual a largura do rio?
   
   
   
   
   
   
  Resposta  1760 metros

   
-
Abra sua conta no Yahoo! Mail, o único sem limite de espaço para armazenamento! 

[obm-l] [OFF} E-mail de contado IMPA

2008-04-08 Por tôpico Gustavo Souza
Alguem teria o e-mail de contato do IMPA para me passar por favor?
   
   
  Muito Obrigado

   
-
Abra sua conta no Yahoo! Mail, o único sem limite de espaço para armazenamento! 

Re: [obm-l] Exercicios de Analise 4

2008-04-03 Por tôpico Claudio Gustavo
  Oi Paulo.
Estou respondendo essa mensagem apenas pra agradecer sua iniciativa. Pois 
essas soluções tenho certeza que ajudarão a muitos outros além de mim.
   
Abraços,
  Claudio Gustavo.

Paulo Santa Rita [EMAIL PROTECTED] escreveu:
  Ola Pessoal,

Tenho publicado algumas solucoes de exercicios de Analise retirados do
excelente Livro :

Curso de Analise - Volume 1 - Projeto Euclides - IMPA
11 edicao - 2 impressao
Autor : Elon Lages Lima

Eu nao publiquei nenhuma solucao dos capitulos 1, 2 e 3 porque sao
assuntos que, em geral, o estudante ja viu em cursos anteriores,
principalmente na Graduacao. Alem disso, eles sao bastante simples.
Entretanto, algumas pessoas me escreveram em off e pediram que eu
publicasse 2 ou 3 solucoes desses capitulos. Assim, selecionei 3
exercicios do capitulo 1 e estou publicando agora. Escolhi os que
achei mais interessantes ou/e desafiadore. Seguem as solucoes :




NOTACAO : A letra lambda sera representada nestes exercicios por
m. Os simbolos de uniao e intersecao serao representados
respectivamente pelos prefixos UNI e INTER. Os simbolos  e 
representarao, respectivamente, contem e esta contido. O Simbolo
de pertence a sera representado pela letra E e f_a representa a
letra f com indice a. A barra / representara a expressao tal
que



( EXERCICIO 1.14)

NOTACAO : Seja f : A - B uma funcao. Se Y  B, f(-1)(Y) sera o
conjunto de todos os elementos x E A tais que f(x) E Y. No caso de Y
ser unitario, tal como em Y={b}, f(-1)(Y) sera representado por
f(-1)(b)

ITEM A :

Seja X  A, a E X. Existe b E f(X) / b = f(a) = a E f(-1)(b) = a
E f(-1)( f(a) ). Como f(a) = b E f(X) = f(-1)( f(X) )  f(-1)(f(a))
= a E f(-1)( f(X) ). Assim, a E X = a E f(-1)( f(X) ). Isto
estabelece que X  f(-1)(f(X)), qualquer que seja o X  A, tal como
queriamos demonstrar.

ADVERTENCIA : No meu livro o enunciado esta errado, pois la pede-se
para demonstrar que f(-1)(f(X))  X para todo X  A. Isto e
evidentemente impossivel. Basta considerar a funcao f:{1,2,3} -{4,5,6
} tal que f(1)=f(2)=f(3)=4. Tomado X={1,2} temos que
f(-1)(f(X))={1,2,3}, isto e, f(-1)(f(X)) NAO ESTA CONTIDO em X

ITEM B :

No item anterior, mostramos que se f:A - B e uma funcao qualquer
entao para todo X  A, f(-1)( f(X) )  X. Agora, supondo que f:A-B e
injetiva, mostraremos que vale tambem f(-1)( f(X) )  X. Faremos isso
por reducao ao absurdo.

Com efeito, suponhamos que f(-1)( f(X) ) NAO ESTA CONTIDO X. Nests
caso, existe um a E f(-1)( f(X) ) tal que a NAO PERTENCE a X,
vale dizer, existe b E f(X) tal que b=f(a) mas a NAO PERTENCE a X.
Como b E f(X), existe c E X tal que b=f(c). Assim, existem a e
c, a # c, tal que f(a) = f(c) = b = f nao e injetiva ... ABSURDO
!

Portanto, f:A- B injetiva = f(-1)( f(X) )  X, para todo X  ªA.
Como f(-1)( f(X) )  X vale para qualquer funcao, seja injetiva ou
não, segue que :

f:A- B injetiva = f(-1)( f(X) ) = X
IMPLICACAO 1

Agora, suponhamos que f: A - B e uma funcao e sabemos que f(-1)( f(X)
)=X para todo conjunto X  A. Queremos mostrar que f:A - B e
injetiva.

Suponha que f:A-B não e injetiva. Neste caso, existem a, b E A tais
que a # b e c=f(a)=f(b). Tomando o conjunto X={a} vemos que f(X)={c} e
que f(-1)(f(X))={a,b}, isto e, f(-1)(f(X)) # X ... ABSURDO ! Logo :

f(-1)( f(X) ) = X, para todo X  A = f:A-B injetiva IMPLICACAO 2

As IMPLICACOES 1 e 2 estabelecem que f:A-B e injetiva se, e somente
se, f(-1)(f(X))=X, para todo X  A, tal como queriamos demonstrar.
( EXERCICIO 1.18 )

ITEM A :

Claramente que Xm  UNI Xm, qualquer que seja m. Aplicando a
propriedade da funcao f, teremos : f(Xm)  f(UNI Xm), qualquer que
seja o m. Assim como todo f(Xm) contem f(UNI Xm) entao :

INTER f(Xm)  f(UNI Xm ). INCLUSAO 1

Por outro lado, e obvio ululante que f(Xm)  INTER f(Xm), qualquer que
seja o m. Aplicando as propriedades enunciadas da funcao, teremos,
sucessivamente :

f( f(Xm) )  f( INTER f(Xm)) = Xm  f( INTER f(Xm)) qualquer que
seja o m =
UNI Xm  f( INTER f(Xm)) = UNI Xm  f(INTER f(Xm)) =
f(UNI Xm)  f( f (INTER f(Xm))) = f(UNI Xm)  INTER f(Xm) =

INTER f(Xm)  f(UNI Xm) INCLUSAO 2

As INCLUSOES 1 e 2 estabelecem que f(UNI Xm) = INTER f(Xm), como
queriamos demonstrar.


***


ITEM B :

Claramente Xm  INTER Xm, qualquer que seja o m. Segue, da
propriedade da funcao, que f(Xm)  f(INTER Xm), qualquer que seja o
m. Portanto :

UNI f(Xm)  f( INTER Xm) INCLUSAO 1

Por outro lado, e obvio ululante que f(Xm)  UNI f(Xm), qualquer que
seja o m. Daqui, aplicando sucessivamente as propriedades da funcao,
vem :

f(f(Xm))  f(UNI f(Xm)) = Xm  f(UNI f(Xm)) qualquer que seja o m =
INTER Xm  f(UNI f(Xm)) = f( INTER Xm)  f ( f(UNI f(Xm))) =
f( INTER Xm )  UNI f(Xm) =

UNI f(Xm)  f( INTER Xm) INCLUSAO 2

As INCLUSOES 1 e 2 estabelecem que UNI f(Xm) = f(INTER Xm), como
queriamos demonstrar.








( EXERCICIO 1.21 )


Dada uma funcao f E F(A;F(B;C)) qualquer. Entao f e uma funcao f:A
- F(B;C), isto e, qualquer que seja o elemento a E A, existe uma
funcao f_a E F(B;C) tal que f(a) = f_a. Como f_a E F(B;C) entao f_a
e uma

[obm-l] Dúvidas - Transformações Lineares

2008-04-01 Por tôpico Claudio Gustavo
  Estou com umas dúvidas numas questões do Apostol pra demonstrar que são 
transformações lineares e dar o posto e nulidade. Os exercìcios sâo do vol.1, 
pag 582.
  25) Seja V um espaço linear de todas as funções diferenciáveis no intervalo 
aberto (-1,1). Se f pertence a V, g=T(f) significa que g(x)=x.f '(x), para todo 
x em (-1,1).
  26) Seja V o espaço linear de todas as funções reais contínuas no intervalo 
fechado de a a pra b. Se f pertence a V, g=T(f) significa que:
  g(x) = (integral de a até b de) f(t)sen(x-t)dt.
Com: a=x=b.
   
Na resposta ambos são transformações lineares.
Acho que não deve ser muito complicado não... 
Obrigado.
Abraços,
  Claudio Gustavo.

   
-
Abra sua conta no Yahoo! Mail, o único sem limite de espaço para armazenamento! 

[obm-l] Méidas

2008-03-29 Por tôpico Gustavo Duarte
 1)Em méidas é fácil mostrar que para 2 valores positivos ,o quadrado da média 
geométrica é igual ao produto das médias harmônicas e aritmética, parém essa 
relação vale tb para 3 ou mais números ? 
 2) faz sentido média ponderada com alguns pesos negativos ?
desde já obrigado.

  1   2   3   4   >